Jump to content

Wikipedia:Reference desk/Science

From Wikipedia, the free encyclopedia

This is an old revision of this page, as edited by Sir Link (talk | contribs) at 11:01, 27 December 2012 (decline of space expansion and exploration). The present address (URL) is a permanent link to this revision, which may differ significantly from the current revision.

Welcome to the science section
of the Wikipedia reference desk.
Select a section:
Want a faster answer?

Main page: Help searching Wikipedia

   

How can I get my question answered?

  • Select the section of the desk that best fits the general topic of your question (see the navigation column to the right).
  • Post your question to only one section, providing a short header that gives the topic of your question.
  • Type '~~~~' (that is, four tilde characters) at the end – this signs and dates your contribution so we know who wrote what and when.
  • Don't post personal contact information – it will be removed. Any answers will be provided here.
  • Please be as specific as possible, and include all relevant context – the usefulness of answers may depend on the context.
  • Note:
    • We don't answer (and may remove) questions that require medical diagnosis or legal advice.
    • We don't answer requests for opinions, predictions or debate.
    • We don't do your homework for you, though we'll help you past the stuck point.
    • We don't conduct original research or provide a free source of ideas, but we'll help you find information you need.



How do I answer a question?

Main page: Wikipedia:Reference desk/Guidelines

  • The best answers address the question directly, and back up facts with wikilinks and links to sources. Do not edit others' comments and do not give any medical or legal advice.
See also:


December 21

Ozone resupply

Is it feasible to fix or mitigate ozone depletion by dispersing artificially produced ozone?--93.174.25.12 (talk) 09:44, 21 December 2012 (UTC)[reply]

Yes and no. Clearly we can (and do) produce ozone. That "metallic" smell you get when a cheap electric motor is running is ozone produced by electrical arcing. But there are two problems with using this to mitigate the depletion of earth's ozone layer:
  1. Ozone is only a good thing at very high altitudes (20 to 30km). Down here at sea level, it's a nasty pollutant. So it's no use just setting up an ozone factory down here on the surface of the planet. The gas would have to either be transported up there - or made up there in some kind of flying factory. Both are exceedingly difficult things to do.
  2. Producing ozone requires energy - and at the scales required to match the ozone that's naturally produced up there, we'd need an ungodly amount of energy. Right now, the odds are good that we do vastly more damage to the atmosphere from CO2 produced as a by-product of all of that energy usage.
SteveBaker (talk) 14:50, 21 December 2012 (UTC)[reply]
No, ozone is constantly being simultaneously created and destroyed in the stratosphere creating an stable equilibrium. Any added ozone would quickly be destroyed returning to the same equilibrium point. Ozone depletion is caused by the presence of some pollutants in the stratosphere which change the chemistry and shift the equilibrium towards a smaller amount of ozone. Dauto (talk) 14:57, 21 December 2012 (UTC)[reply]
So continuously pump in more and more ozone to keep it away from equilibrium. Whoop whoop pull up Bitching Betty | Averted crashes 23:52, 21 December 2012 (UTC)[reply]
Well it's really more accurate to simply say that the pollutants (or rather their eventually chemically-unbound constituents) simply act to break down the reactive O3. The construct of a stable equilibrium state is an artificial one in this context and confuses the issue a bit. In any event, more to the point, there really isn't much need to produce ozone for this purpose; since the implementation of more significant controls of CFCs, ozone depletion has stabilized and even begun to be reversed in the most potentially ecological hazardous regions. As noted above, there are other pollutants which are more significance issues at present. Snow (talk) 07:56, 22 December 2012 (UTC)[reply]

Eight compartments in the rectus abdominis - which comprise the six pack?

So I'm not entirely sure whether it's rows 1-3 or rows 2-4 of the rectus abdominis that count as the six pack. By that I mean is the pair of somewhat angled compartments at the top of the muscle considered part of the six pack (or does it have some other colloquial name?) or is the pair of long compartments at the bottom of the muscle - below the navel - the bottom of the six pack (if no then does it have some other colloquial name?)?

http://robertsontrainingsystems.com/wp-content/uploads/2012/04/rectus-abdominis.jpg 184.152.17.125 (talk) 19:20, 21 December 2012 (UTC)[reply]

Looks like it's the top row, to me: [1]. The bottom row tends not to be as visible. StuRat (talk) 04:17, 22 December 2012 (UTC)[reply]
Well, given that "six-pack" is a colloquial term and not a clinically anatomical one, it really has no absolute definition of which superficial features of the abdomen belong to it. That being said, in a person with well-defined-enough musculature, the outline of all of segments of rectus abdominis would be obvious, though which would be more prominent would, of course, vary by individual. I don't know that people are really used to defining the "six" of the six-pack so precisely so much as using the expression idiomatically to refer to the whole of the mid-abdomen in a fit person, but I suspect if you asked people to make a call, they would mostly select rows 1-3 as the main constituents. Snow (talk) 08:07, 22 December 2012 (UTC)[reply]

Wireless energy and magnetic field

So, I was watching this video documenting wireless energy and how it would use magnetic fields to get the energy from one object to another. In it described some concerns with safety (e.g. cancers), and someone briefly described how we've been bathing in a magnetic field for thousands of years anyway, so there would be no real issue.

Though, two questions come to mind: would the magnetism from the wireless energy "outputs" be much higher than the Earth's? And second, why not just harness the magnetic field that we've been living in? Thanks! 174.93.61.139 (talk) 21:13, 21 December 2012 (UTC)[reply]

If you look at Orders of magnitude (magnetic field), you'll see that the earth's magnetic field is incredibly weak. (25 to 65 micro-tesla). By comparison, a small refrigerator magnet is about 200 times more powerful and a small neodymium magnet (such as you find in many modern products) could be around 100,000 times stronger. So while we're pretty sure that the earth's magnetic field is "safe" - it's by no means obvious that much higher magnetic fields would not have health risks. SteveBaker (talk) 21:44, 21 December 2012 (UTC)[reply]
A steady magnetic field does not transmit energy. It has energy, but the energy just sits there. So there is really nothing to "harness" from the Earth's magnetic field.
To transmit energy, what you need is a changing magnetic (and electric) field; that is, you need electromagnetic radiation. Such radiation has known, and probably some unknown, health effects, all of which vary widely depending on frequency/wavelength, but which are in general quite distinct from (and likely to be more serious than) any dangers from a static magnetic field (of the same average intensity as the changing one, say). --Trovatore (talk) 00:23, 22 December 2012 (UTC)[reply]
As for known health risks, those with metal implants must avoid strong magnetic fields, while extremely powerful magnetic fields can interfere with anyone's blood flow by affecting the iron in hemoglobin. StuRat (talk) 04:11, 22 December 2012 (UTC)[reply]
You wanna pop up a cite for that last statement? Ferromagnetism as I understand it is a bulk property of iron; it isn't usually seen in iron compounds (except maybe magnetite), where the iron atoms are dispersed from one another. (I don't doubt that sufficiently powerful magnetic fields are dangerous, but I'm skeptical that the mechanism you propose is a particularly important one.) --Trovatore (talk) 06:08, 22 December 2012 (UTC)[reply]
Perhaps StuRat believes in Magnet therapy? The Science ref-desk is no place to spread fringe theories as if they had a scientific basis. DMacks (talk) 06:21, 22 December 2012 (UTC)[reply]
No, I certainly do not. While Magnet_therapy#Purported_mechanisms_of_action states that blood is weakly paramagnetic, the magnets used in "magnetic therapy" are nowhere near the strength needed to have any effect, and, even if they were, how would interfering with the normal flow of blood be helpful ? StuRat (talk) 06:25, 22 December 2012 (UTC)[reply]
Some relevant articles: Electromagnetic radiation and health, Wireless electronic devices and health, Mobile phone radiation and health. As far as I understand, for common household devices (mobile phones, wifi routers,...) there are many studies, but no generally accepted view, also due to the fact that people are exposed only for a couple of years, making the discovery of long-term effects difficult. As the situation is not clear, personally I'd minimize exposure, by switching of wireless devices when not needed, not sleeping near wireless sources, etc. bamse (talk) 08:52, 22 December 2012 (UTC)[reply]
There's an awfull lot of rot published about the effects of electromagnetic radiation, and cell phone emission in particular. The simple fact is this: for any medical/physiological effect at all, at least one of two things must be satisfied: a) the field strength is so intense that heating of body parts occurs above that which the bloodstream (acting as a heat conductor) can cope with. This requires close proximity to quite high power radio transmitters well and truely beyond what a cellphone can put out. And the output of wifi routers and other local area wireless devices is even more minute. The stray radiation from tube-type TV sets is considerably greater and nobody worried about them, not should they. b) the radiation is of sufficiently high frequency to be ionising radiation, eg X-rays. Cell phone and wireless device radiation is too low in frequency to be ionising. If neither of these two conditions are satisfied, then chemical bonds cannot be broken, and if chemical bonds are not broken then there cannot be any effect on the body. End of story. The trouble lies in how research grants are granted, and how editors select articles for publication. If yoiu are a researcher, and ask for a grant to show that everything is hunky dory and nobody should worry, a grant committee, who probaly don't really understand it anyway, will not give you a grant. But if you write out an application stating that a possible hazard needs to be investigated, you may get a grant. And if you beaver away spending teh grant money and finally conclude that while you could not definitely establish cause and effect, there should be more research done, well, you might get more grant money. And editors want to sell journals, so even in a professional journal, a bit of sensationalism helps. Ratbone 124.182.16.117 (talk) 10:33, 22 December 2012 (UTC)[reply]
OK, first of all, field strength per se doesn't heat anything. A static field just sits there and transmits no power; that was the first point I made in the first response to the question. Nevertheless, a sufficiently strong static field can indeed have effects on body functioning (things with dipole moments line up with the field; at some point that's going to affect something).
None of that is very applicable to cell phones, but it does indicate a problem with the line of reasoning that starts there are only two mechanisms for such and such.... Just because you haven't thought of a mechanism doesn't mean there isn't one. I personally am not very worried about cell phones, but sufficiently rigorous epidemiological evidence would change my mind, whether or not I understood the mechanism. --Trovatore (talk) 21:43, 22 December 2012 (UTC)[reply]
Re static (steady) fields - yest they are irrelevant to this discussion - because you cannot have a "static" electromagnetic wave. You can have steady magnetic fields and steady electric fields, but to get an electromagetic wave, you must have time-varying fields. Your statement that just because I haven't thought of a method doesn't mean that there isn't one looks ok in logic. However, the requirement for either ionising radiation or sufficient intensity for heating is well established in science. Beliveing that there may be some unknown 3rd method waiting to be discovered is much like believing that there my be a way of teleporting Star Trek style just waithing to be discovered. It would be contrary to known scientific facts. Ratbone 124.182.144.218 (talk) 01:24, 23 December 2012 (UTC)[reply]
The original question appeared to be about static fields. The questioner seemed to think they could be used for power, which of course they can't, but they were part of the question so they're not irrelevant.
Yes, but I was correcting the misconception posted by Bamse, involving radio emissions from cellphones, wireless routers and the like. Ratbone 124.182.144.218 (talk) 02:07, 23 December 2012 (UTC)[reply]
The rest of your statement is wrong. There are known effects of static magnetic fields, and they do not heat anything nor break any bonds. Even if there were not any known ones, philosophically your tack is just plain wrong. It is perfectly good science to look for an effect even when no mechanism is proposed. --Trovatore (talk) 01:29, 23 December 2012 (UTC)[reply]
No, its only good science to look for effects when there is at least a plausible theory, that cannot be rejected by known facts, that suggests that looking will be worthwhile. Otherwise time would be wasted. One might as well run tests to look for effective telepathy. That was a reasonable thing to do at one time, but not now that the electromagnetic sprectrum is understood. Can you nominate a known effects of radio emission of living animals that does not involve breaking chemical bonds? I think not - simply because any effect MUST involve chemical bonds - we are after all, made up of atoms combined together - we are a complex chemistry set. Can you even cite a plausible theory for an effect? Ratbone 124.182.144.218 (talk) 02:07, 23 December 2012 (UTC)[reply]
You have a cramped and restrictive view of science, one that is in my personal opinion very very wrong indeed. It is in fact good science to look for telepathy, and it has nothing to do with whether the electromagnetic spectrum is understood. It so happens that well-designed experiments don't tend to find it, but that doesn't mean it wasn't good science to do them in the first place.
So no, I don't have to cite a plausible mechanism in order to look. If you find the effect, then you look for the mechanism.
HOWEVER, all that aside, static magnetic fields (which, recall, is what we were originally talking about) have documented effects on living organisms, without breaking any bonds. --Trovatore (talk) 02:44, 23 December 2012 (UTC)[reply]
There have been some studies that show the area around the ear where the cell phone is held gets a bit warmer. However, I don't see any need to resort to EMF explanations. The phone simply acts as a thermal insulator and reflects heat back onto the ear which otherwise would be radiated away. StuRat (talk) 19:12, 22 December 2012 (UTC)[reply]
Carefull, Stu. Somebody might think that warming the ears is itself bad Some of the warming will be due to absorbing radio frequency energy as the blood flow to the ears and nearby scalp is not great, so heat is not conducted away very well. This does not apply to the brain as brain blood flow is quite prodigious. However, nobody has reported any problems wearing earmuffs and full helmets and nobody wants to investigate them. And we have other dangly bits that in the caveman days probably remained pretty cold but now sit within our clothes at close to body temperature. Appart from possibly the lower sperm count in modern man (which has not been explained), no problems have been reported. Ratbone 124.182.144.218 (talk) 02:45, 23 December 2012 (UTC)[reply]
This is exactly my point. Studies showing warming on the side of the head near the cell phone might incorrectly portray this as a "sign of dangerous radiation", when it's really just the result of having something warm next to your head. StuRat (talk) 07:54, 23 December 2012 (UTC)[reply]
Apart from heating and ionization, could the radiation affect electrical signals in neurons/the brain? bamse (talk) 20:24, 22 December 2012 (UTC)[reply]
The short answer is essentially no. The signalling between and within neurons is electrochemical. This means that the operation of the brain and nervous system can be interfered with by electric fields - as in electric shocks. The function can be affected without altering the structure. However, it has been well established for 100 years or more that the functional sensitivity to electric fields is inversely proportional to frequency. At the frequency of the power mains (50Hz or 60Hz depending onm country) the sensitivity is very marked - currents as little as a few microamps within the brain will produce conscious effects. AT 440 Hz (A above middle C) the sensitivity is reduced by a factor of about 100x. At 20,000 Hz (the upper limit of human hearing), sensitivity to electric current is practically gone completely. The frequencies used by cellphones and routers are well and truely beyond these frequencies. Technicians who work in radio broadcast transmitters (which use frequncies well below that of cellphones and wifi routers) get taught in their training, and sometimes learn the hard way, that coming into electrical contact, or experincing high field strengths with high frequency energy cannot be felt other than the normal pain felt from the resulting burns. Ratbone 124.182.144.218 (talk) 02:21, 23 December 2012 (UTC)[reply]

Regarding the Stu's suggestion that a strong enough magnet could affect hemoglobin, as may also be aware, anything made out of atoms can be moved through direct diamagnetic levitation. The magnet has to be really strong though, and is not something that would fit on your wrist. Someguy1221 (talk) 11:16, 22 December 2012 (UTC)[reply]

Could the hand have a more ideal anatomy?

Ignoring any sentimental feelings we might have for the current configuration, or it feeling unnatural if you were suddenly given it (if you were born with 4 fingers and 2 thumbs, those hands you can control like second nature and the stock one would feel wrong) What could we do to better serve 21st century first world humans, more than hunter-gatherers? How about more fingers? Longer ones? Thinner ones? Extra joints? Of course extra length means the palm has to be longer to close them/make fists. More fingers probably means thinner and therefore weaker and easier to break. And forget opposable everything, there's no room for enough muscles. Really, I want longer thumb, though. (Added bonus: if we have 12 fingers, we get to use the superior base-12) Sagittarian Milky Way (talk) 21:45, 21 December 2012 (UTC)[reply]

The problem is that we humans modify our environment to suit our bodies - and on a longer timescale, evolution has modified our bodies to suit our environment. If our hands were larger, our cellphones would have bigger touch-screens. The tools we use like screwdrivers are optimized to produce the maximum amount of torque for the hands we have. Everything we do is built on a scale where we can interact with it. So I suspect that most changes that you could imagine would be worse than what we have. Sure, you could get more dexterity at the cost of less strength - or vice-versa - you could imagine hands that were more able to cup liquids or hands with a hard pad to allow us to use them as hammers. But for every improvement you could come up with, there would be some kind of downside - because we're adapted to our environment - and vice-versa. SteveBaker (talk) 21:57, 21 December 2012 (UTC)[reply]
Clearly, some of these things might not have realistic evolutionary paths. And you can't seperate cause and effect completely, but if intelligent designers had adapted us for this technological level instead of cave-man, our chimp-men ancestors might've had a somewhat harder life but we'd have an easier one and our technology would be suited to that (different game console controllers and screwdrivers maybe?) I don't know about the hammer pad though, rocks are everywhere. And even hand bone can get injured from punching a forehead hard enough, much less nails. But screwdrivers are not optimized to produce the maximum amount of torque for the hands we have. Assuming enough room, that would be something like this or that:
. Sagittarian Milky Way (talk) 22:39, 21 December 2012 (UTC)[reply]
I've thought having a second thumb in place of the pinky might improve dexterity. StuRat (talk) 04:06, 22 December 2012 (UTC)[reply]
When working inside PC cases and around car engines I've often wanted a longer thumb, and sometimes finger joints that bend both ways. HiLo48 (talk) 04:11, 22 December 2012 (UTC)[reply]

This new study suggests ours hands evolved as weapons, at least in part. Zoonoses (talk) 06:21, 22 December 2012 (UTC)[reply]

So I can't have longer thumbs just because they might stick out too far when you punch on instinct (i.e. without setting) and get hurt? Typical human. Sagittarian Milky Way (talk) 00:45, 24 December 2012 (UTC)[reply]

I suspect quite a few pianists would nominate being able to stretch further. Double sharp (talk) 13:55, 24 December 2012 (UTC)[reply]

If we started all over again, and ignoring that humans like to use the number of fingers they have, which number base is best to use?

12? 16? 20? If we go too high (like the Babylonians' base-60) there are too many symbols and practical scale numbers of the current era which aren't outrageously long in the first place (US national debt, 7 billion people) don't get much shorter. Lets say we don't wan't to just have base 26 (or 25, 24) by appending a mark to each letter. There are enough umalauts and diacritic marks in the world already. Sagittarian Milky Way (talk) 21:59, 21 December 2012 (UTC)[reply]

My opinion, first of all, is that base 12 is overrated. Sure, you can more easily divide by three. How often is that a big issue? It's not that hard in base ten. And in base 12, you can't divide as easily by five.
Too small a base and numbers become unwieldy; too large a base, and you have to memorize big addition and multiplication tables. Other than that I don't think it makes that much difference. Base 8 or 16 would have some minor advantages for the man–computer interface. --Trovatore (talk) 22:16, 21 December 2012 (UTC)[reply]
I guess a base 16 multiplication table is a bit large. But clearly, that we feel the compulsion to make base-10 multiplication tables go to 12 and 112 pound "hundreweights" and that Dunbar's number is 150 shows that there is an unmet need for larger two-digit numbers. Now onwards to make the 12-fingered human! Sagittarian Milky Way (talk) 01:13, 24 December 2012 (UTC)[reply]
The Babylonians used a base that could more accurately be described as mixed 6 and 10 so it didn't require all that many symbols. Dmcq (talk) 01:38, 22 December 2012 (UTC)[reply]
From a computational standpoint, the ideal number base would have as many small factors as possible. Base 60, for example, is good: the prime factorization is 2*2*3*5, making division by 2, 3, 4, 5, 6, 10, 12, 15, 20, and 30 easy. From a usability standpoint, the ideal number base would have as few symbols as possible. Binary is ideal, with only two symbols. The "perfect" number base is a balance between the two goals (base 10 is fairly lousy from a computation standpoint, since dividing by 5 (easy) is less common than dividing by 3 or 4 (both hard)). --Carnildo (talk) 01:47, 22 December 2012 (UTC)[reply]
That doesn't seem right, unless "usability standpoint" is from the point of view of a computer. In human terms, binary is horrible to use. 86.171.174.84 (talk) 22:49, 22 December 2012 (UTC)[reply]
Also, in the earliest stages, there weren't distinct numbers, but rather a series of repeated objects. So, in a base-10 system, for example, 9 dots followed by a space, followed by 6 dots, followed by a space, followed by 2 dots would represent the number 962. In such a system, it's best to use a small base, limited by how many dots most people can recognize instantly without having to count them. Always placing the dots in the same pattern, as on dominoes, allows for a somewhat larger number. StuRat (talk) 04:04, 22 December 2012 (UTC)[reply]
Could you provide a cite for that, StuRat? I've seen that system before, but only as a latter day method of displaying the results of an abacus or similar instrument. Such a system would seem hinged on the idea of breaking numbers into "ones", "tens", "hundreds", and so on, which (I thought) was dependent on using an existing decimal system and its use of zero as a placeholder. Matt Deres (talk) 02:19, 23 December 2012 (UTC)[reply]
See Maya numerals (and note that I was only using powers of 10 in my example for simplicity, while the Maya used base 20). StuRat (talk) 06:20, 23 December 2012 (UTC)[reply]
That's very interesting; thanks for the link. Matt Deres (talk) 14:47, 24 December 2012 (UTC)[reply]
You're welcome. StuRat (talk) 05:30, 26 December 2012 (UTC)[reply]
Dividing by 4? That's not hard in decimal. Multiply by 0.25. All your base. - ¡Ouch! (hurt me / more pain) 18:04, 22 December 2012 (UTC)[reply]


Base 200560490130. Count Iblis (talk) 15:01, 23 December 2012 (UTC)[reply]

Accept these multicolored bitmaps as my proposed symbols for 200560490128200560490130 and 200560490129200560490130 (magnified 0.1 times) > :
👍 Like Double sharp (talk) 13:52, 26 December 2012 (UTC)[reply]
Heh. My illustrations of uniform tilings of the hyperbolic plane (see my Commons page) are 2520 pixels across, because that's lcm(1,2,3,4,5,6,7,8,9,10). —Tamfang (talk) 17:56, 27 June 2013 (UTC)[reply]
Is that because those are the decimal digits? ;-P Why not 27720?
Personally, I would say base 60 is the best. Double sharp (talk) 05:25, 20 November 2013 (UTC)[reply]
Not lcm(1,2,3,4,5,6,7,8,9,10,11,12) = 27720 because I didn't want to wait 121 times as long for my computer to finish the damned things. —Tamfang (talk) 05:47, 20 November 2013 (UTC)[reply]
Diminishing returns of higher primes, eh? ;-) Double sharp (talk) 08:28, 20 November 2013 (UTC)[reply]
Well, slower returns. (It took me over a week to fill that table. Afterward, I noticed that my code used complex arithmetic where it could have used real arithmetic. Oh well.) —Tamfang (talk) 18:15, 20 November 2013 (UTC)[reply]


December 22

Generating electricity from a hypothetical magnetic monopole

Hypothetically, suppose I have a stable magnetic monopole and I want to generate electricity from it, according to the modified Faraday's_law_of_induction from Magnetic_monopole:

Assuming that it's a permanent magnet, then B is constant w.r.t. time and it simplifies to:

According to this, all I would have to do is wound enameled wire around the magnetic monopole to generate electricity, does that sound about right?

No motion is required at all, right?

And it shouldn't require more than one monopole, right?Dncsky (talk) 01:35, 22 December 2012 (UTC)[reply]

is a current, so motion is required. -- BenRG (talk) 07:10, 22 December 2012 (UTC)[reply]
Could you elaborate a bit? What kind of motion would be required? Does the right hand rule still apply here? I was under the impression that is a current flowing out of the monopoly in all directions (to God knows where). I'm having a really hard time conceptualizing a current that doesn't travel in a loop, hence the question. Dncsky (talk) 12:37, 22 December 2012 (UTC)[reply]
No, is the magnetic current generated by the motion of the magnetic charge - analogous but independent from the electric current generated by the motion of electric charges. Dauto (talk) 12:53, 22 December 2012 (UTC)[reply]
I see. So would a generator built using magnetic monopoles be any more efficiency than one built using magnetic dipoles? Or would their efficiency be the same? Dncsky (talk) 13:26, 22 December 2012 (UTC)[reply]
There is no reason to think they would be more efficient. Dauto (talk) 13:42, 22 December 2012 (UTC)[reply]
Thank you. I got this out of a SF story. Guess the author didn't do his research.Dncsky (talk) 14:43, 22 December 2012 (UTC)[reply]
There may be an interesting twist of interest though: Use of a material that conducts magnetic monopoles (just like a wire conducts electrons) in place of the magnetic core would presumably allow a transformer to operate at DC, not only AC. Perhaps an interesting application for spin ice? — Quondum 10:30, 27 December 2012 (UTC)[reply]

Dry ice fog

The article on dry ice is rather weak on the detail of how dry ice + water = clouds of "smoke". I gather the "smoke" is condensed water vapour, but does this come from the surrounding air or from the water? What is the exact mechanism of its production? Also, why does the "smoke" sink? Is it because carbon dioxide is heavier than air (as I read in one place) or because the vapour is cold (as I read in another)? — Preceding unsigned comment added by 86.160.86.1 (talk) 03:09, 22 December 2012 (UTC)[reply]

Most likely this is the same phenomenon as sea smoke, which occurs when very cold air (or in this case, CO2) lies over warmer water. From the article, Evaporation from exposed water surface depends upon its vapor pressure. If the water temperature is greater than that of the nearby air, the evaporation continues faster than the air can absorb the water vapor, even though the cool air's relative humidity is 100%. This further evaporates immediately and re-condenses as visible fog.
The reason why the "smoke" sinks, is that the ice crystals in the "smoke" are heavier than the air/CO2 mixture. --NorwegianBlue talk 15:51, 22 December 2012 (UTC)[reply]

why charcoal and salt should be used around the earth conductor in the earthing pit — Preceding unsigned comment added by 188.135.7.2 (talk) 09:03, 22 December 2012 (UTC)[reply]

What sort of earthing do you have in mind? Electrical HV distribution earthing? Domestic residence electrical earthing? Radio station earthing? Telecommunications (phone company) eathing? Earthing for other purposes such as corrosion control? I have been involved in earthing for HV substations, radio stations, and phone company infrastructure, and I have not seen nor heard of charcoal being used. I have not seen, but have heard of salt being suggested, but it is not generally used, as it will give only a temporary improvement.
The main purpose of added substances around electrical earthing electrodes in the ground is to improve conduction to earth by making the surrounding matetial hygroscopic, as the mineral components of sand and soil are electrical insulators. Electrical conductivity in sand and soil is provided by interstitial water (in desert areas with very little moisture in the soil, getting a decent earth is extremely difficult, at least unless you can drive earth electrodes down into the water table, which, if the soil is dry, will probably mean drilling down great depths and going thru a rock layer). A secondary factor in some areas may be to reduce soil chemical action/corrosion of the earth electrode(s). Bentonite is the usual material used. Sometimes a small amount of salts in a proprietry mix is added to the bentonite when the bentonite is sold specifically for eathing purposes, and is claimed by suppliers to improve conduction still more. However, any benefical effect must be short term, and in any case it can be shown by mathematical analysis that any such improvement over damp bentonite is marginal anyway. The improvement for added salts comes from improving conduction in the water in teh soil/bentonite, but any added salts, not already in the soil, will gradually leach away until diffusion equilibrium is reached. Conversely, if you back fill with pure bentonite or sand, over time salts and other conduction-promoting substances in the soil will gradually diffuse in. So if your electrical earth is good enough electrically upon installation, over time it will typically get a bit better, providing you do NOT add salts.
With radio transmitter earthing, usually no soil treatment is done, as it is more effective to just bury sufficient radial wires to make a ground plane.
With domestic electrical earthing in the Multiple Earth Nuetral system, no soil treatment is done. The safety and operation of the MEN system does not depend on any single residence earth - it depends on a multitude of earths working together, so the performance of any one earth is unimportant.
You will get a better answer if a) you make your question more specific, and b) you append your question with a pen-name. Its a free world in Wikipedia, and you don't have to, but you will get better answers if you do. Keit 121.221.223.13 (talk) 10:01, 22 December 2012 (UTC)[reply]
Also, in case you heard that pure water is an insulator, and are thinking that the salt was added for that reason; I don't think so. Yes, pure water is an electrical insulator, but you won't encounter water that pure in the ground, so it's not an issue here. StuRat (talk) 19:06, 22 December 2012 (UTC)[reply]
Googling "charcoal earthing" returns some websites discusing it, but they all seem to be posted question-and-answer type sites of very dubious value. Charcoal will be more costly than bentonite and can be shown by analysis that even assuming the charcoal is simply a graphite cylinder around the earth electrode, any improvement over damp bentonite will be slight. Wickwack 120.145.48.199 (talk) 14:14, 24 December 2012 (UTC)[reply]

Scratching a cat's spine

There's a spot maybe 3/4 the way down the spine, extending to the base of the tail, on at least some house cats, that, when scratched, causes the cat to behave very strangely, like having an epileptic fit or an orgasm (making jerking, repetitive motions and noises). I imagine there's a nerve bundle there, but:

1) What precisely is it called ?

2) Why does it have this effect ?

3) Do all cats behave this way ?

4) Do other felines ?

5) Do other animals ? StuRat (talk) 19:00, 22 December 2012 (UTC)[reply]

My first thought is that it's related to female sexual activity, in that, in my cats at least, scratching this spot produces lordosis. Good question and I shall have fun trying to track this down. Thanks. --TammyMoet (talk) 19:31, 22 December 2012 (UTC)[reply]
Don't have too much fun... 86.171.174.84 (talk) 20:23, 22 December 2012 (UTC)[reply]
I think Tammy is on to something, but (WP:OR) some male cats do it too, and certainly not all cats give a noticeable reaction. I've also seen some dogs act weirdly when scratched there, but it is more of a ticklish type response. SemanticMantis (talk) 22:23, 22 December 2012 (UTC)[reply]
The only cat of mine that has a strong response in this matter is a male - scratching here makes him flop over and show me his belly. I never understood why. Someguy1221 (talk) 23:48, 22 December 2012 (UTC)[reply]
Well, something odd definitely happens involving that spot, whatever the behavior exhibited by each particular cat. StuRat (talk) 01:45, 23 December 2012 (UTC)[reply]
Our cat (gib), responds by standing on his hindlegs and resting his front on the ground, as if he can't decide whether to stand or lie down, so he ends up doing both concurrently. It seems to be an enjoyable experience for this particular individual, other cats dislike this type of attention, and will respond aggresively after a period of endurance. Plasmic Physics (talk) 05:49, 23 December 2012 (UTC)[reply]
One of my cats, male, has gone through periods of hypersensitivity, where if you touch that spot he will compulsively lick his front paws. It seems to cause him some distress so I try not to do it. He hasn't seemed to have the problem recently; I don't know what makes the difference (could be that I've been more diligent about treating him for fleas). --Trovatore (talk) 05:53, 23 December 2012 (UTC)[reply]

While we are on the topic of cats, why does our cat fetch one of us to stand around while it feeds? Plasmic Physics (talk) 07:53, 23 December 2012 (UTC)[reply]

Because it can! --TammyMoet (talk) 10:26, 23 December 2012 (UTC)[reply]
You may be standing around but the intention may be to have you join the meal. Bus stop (talk) 20:11, 24 December 2012 (UTC)[reply]
That is silly. Just because I can pat my head and rub my stomach, does not mean that I do it on a daily basis. Plasmic Physics (talk) 20:38, 23 December 2012 (UTC)[reply]
Animals can be a bit vulnerable while eating, both because they are distracted and because the "kill" may attract unwanted attention from other animals out to steal their prey and maybe kill them in the process. So, in social animals, having one stand guard while another eats makes sense. Cats aren't naturally social (although they are related to lions, which are), but seem to have taken on social aspects when they were adopted by people. StuRat (talk) 22:57, 23 December 2012 (UTC)[reply]
Some cat species are social, however. They may still hunt alone, but they will eat and sleep as a group. Someguy1221 (talk) 20:19, 24 December 2012 (UTC)[reply]
Which cat species ? StuRat (talk) 20:21, 24 December 2012 (UTC)[reply]
Lions, for instance. I have also consistently found feral housecats moving in pairs (same pair repeatedly for over a week), but these may be ephemeral mating pairs. The are anecdotal reports of large feral "colonies" being found by animal control. Someguy1221 (talk) 20:27, 24 December 2012 (UTC)[reply]
To answer the OP, I found an article which says that it's "an invitation to come closer and interact" when done in the context of human/feline interaction, but that suggests it's initiated by the cat, whereas I see it almost always initiated by the human. --TammyMoet (talk) 10:29, 23 December 2012 (UTC)[reply]
That describes what lordosis is, but not what the spot on the spine is. StuRat (talk) 19:34, 24 December 2012 (UTC)[reply]

Perceiving numbers

Are there any scientific studies on how people perceive different numbers which are essentially the same?

1) For instance, 2 kg vs. 2000 g, or 1 Euro vs. 100 Yen.

2) Related, but slightly different, would a European perceive the difference between 1 and 2 Euro in the same way as Japanese the difference between 100 and 200 Yen?

3) Third question: Would a European perceive the difference between 1 and 2 Euro in the same way as same European the difference between 100 and 200 Yen? bamse (talk) 20:18, 22 December 2012 (UTC)[reply]

I numbered your Q's for ease of responders:
1) I believe a larger number universally makes people think of a larger quantity, regardless of the unit. So, 2000 g sounds like more than 2 kg. StuRat (talk) 20:48, 22 December 2012 (UTC)[reply]
StuRat's unreferenced beliefs aside, the place to look would be market research studies, as product manufacturers are intensely interested in which fashion would be preferred by customers. Someguy1221 (talk) 05:19, 23 December 2012 (UTC)[reply]
I agree with Stu, I certainly have that bias, the same way that I imagine a sour taste when I watch someone eat a lemon. I'm also biased towards a higher level of precision, regardless of whether that is true. I have to convince myself that I don't have the information neccessary to differentiate between the two. Plasmic Physics (talk) 05:37, 23 December 2012 (UTC)[reply]
Yes, and it's the reason why salesmen don't volunteer to tell you the full price, but just what an installment costs. Many people don't do the math, and lower numbers sound better. StuRat (talk) 06:03, 23 December 2012 (UTC)[reply]
I'm not sure this is true. I can certainly remember beauty adverts which boast that users of the product will see the difference in just "14 days", rather than "2 weeks", possibly because the former 'feels like' a shorter length of time. --Iae (talk) 11:26, 23 December 2012 (UTC)[reply]
Not precisely the same thing, but related is Psychological pricing. Vespine (talk) 02:17, 24 December 2012 (UTC)[reply]
Any number of days seems shorter than any number of weeks or months, even if the first number is way higher than the second number. Being sentenced to 60 days in prison somehow feels not too bad; but even 1 month in prison - hell, that's a long time. In these cases, the unit of measurement outweighs the actual numbers involved. Who says humans aren't weird? -- Jack of Oz [Talk] 01:05, 25 December 2012 (UTC)[reply]
  • The radio show "Radiolab" just aired an episode this week dealing with this exact subject. See http://www.radiolab.org/ and select the audio for the most recent show, titled "Numbers". The first segment, I believe, was about the science behind people's number sense, and was quite facinating. --Jayron32 03:18, 24 December 2012 (UTC)[reply]

pipes history

I am interested in the early history of pipe manufacture. In particular, how were the first wooden pipes made? how were the first metal pipes made? Were there any related products that used the same manufacturing techniques around during their invention? 204.191.89.147 (talk) 20:37, 22 December 2012 (UTC)[reply]

The first wooden pipes probably didn't need to be manufactured at all. Bamboo, for example, forms hollow pipes naturally, so you can just pick up old bamboo off the ground: [2]. Other trees aren't quite so helpful, but the bark of some might form a nice tube, although with a split where it was removed from the trunk (they'd also need to be quite flexible to avoid breaking elsewhere). I imagine these type of pipes were used long before it became possible to drill holes through a solid trunk. StuRat (talk) 20:52, 22 December 2012 (UTC)[reply]
As for the earliest metal pipes, one way might be to pour molten metal (say copper) into a vertical cylindrical form (perhaps made of sand). If you then waited the proper length of time, only the outside would have hardened, and you could drain the inside out, although you'd need to keep the ends heated so they didn't harden. Pipe like this wouldn't be all that good, but might work to line an existing masonry sewer, preventing a collapse. StuRat (talk) 21:04, 22 December 2012 (UTC)[reply]
Anything you wanted to know about early water pipes at Sewerhistory.org. Isn't the internet a wonderful thing? Alansplodge (talk) 23:05, 22 December 2012 (UTC)[reply]
thanks for the link. That one looks promising. 204.191.89.147 (talk) 01:07, 23 December 2012 (UTC)[reply]
Bamboo was not available to most of the world. I suspect the first pipes were made in the middle east and africa. Also, sand casting is plausible, but puts the time of invention later than my first guess. 204.191.89.147 (talk) 01:07, 23 December 2012 (UTC)[reply]
Based on what I've read from that link, it appears that clay predates metal pipes (which is so obvious when you think about it), and that at least some early pipes were cold forged then soldered. That was a really useful link. 204.191.89.147 (talk) 02:05, 23 December 2012 (UTC)[reply]
Yes, I hinted at that with my comment on "existing masonry sewers", but I should have been more explicit. Can we mark this Q resolved ? StuRat (talk) 06:54, 23 December 2012 (UTC)[reply]
I'd like to also add a link to our (unreferenced) Tap_water#History. It is claimed that pipes made of clay and straw were found in Mesopotamia circa 3000BC, and brass and copper pipes in Egypt in 2500BC, although the means of manufacture is not given. The Romans made their pipes out of lead sheet metal, shaping the flat sheets into the desired form. Someguy1221 (talk) 07:04, 23 December 2012 (UTC)[reply]
Resolved

Your wish is my command. Alansplodge (talk) 17:10, 23 December 2012 (UTC)[reply]

I have seen some wooden piped made in the early 19th century. They were produced from long logs by a boring process. Yhen the end of one was trimmed down like a pencil to fit into the end of the next. 99.140.254.238 (talk) 19:51, 24 December 2012 (UTC)[reply]

At the top of the article, it says "invariant mass" and "rest mass" are the same thing, I also looked up on several other websites that says the same thing. What confusing is, there is a section in the article devote to explain about difference between invariant mass and rest mass. If they are the same thing then how could there be an entire section telling their difference? After read the section, I still have no idea what are they talking about. Apparently, the wordings are not for someone who doesn't know much about physic to understand it.184.97.227.164 (talk) 21:43, 22 December 2012 (UTC)[reply]

They are identical when both can be defined. The difference is that when you're talking about a system of particles - as opposed to a single particle - it may not be possible to define rest mass since the particles may be moving with respect to each other. The invariant mass on the other hand can always be defined. Dauto (talk) 23:14, 22 December 2012 (UTC)[reply]
Usually,AFIK, if one can be defined so can the other, thus an example where one cannot be defined would help. --Trillianthcircuit (talk) 23:49, 22 December 2012 (UTC)[reply]
I think it's a language thing — to some people it seems odd to refer to the "rest mass" of something that, no matter what reference frame you choose, still has parts of it moving. --Trovatore (talk) 00:03, 23 December 2012 (UTC)[reply]
Invariant mass and rest mass are the same thing. A beam of light, which can't be at rest, is routinely said to have a rest mass of zero. The "invariant mass vs. rest mass" section is wrong to make a distinction and needs to be changed. -- BenRG (talk) 00:34, 23 December 2012 (UTC)[reply]
Well, how about a rotating object? I think most people would intuitively understand its "rest mass" to be the mass after you stop its rotation, but its invariant mass is larger than that. --Trovatore (talk) 00:38, 23 December 2012 (UTC)[reply]
That isn't a good example. The invariant mass isn't larger, because, if I'm not mistaken, you need to subtract the energy due to the rotation to obtain the object's invariant mass. I'm not certain, but I think Ben is correct and the article needs a fix. -Trillianthcircuit (talk) 19:13, 24 December 2012 (UTC)[reply]
OK, I know this section has been archived now, but I think it's important to respond to this for the benefit of anyone who may be searching later. I'm afraid you are mistaken. The invariant mass is equal to the energy (divided by c2, if you're not using natural units) in the center-of-momentum frame; that is, the inertial frame in which the total momentum is zero. In that frame, the object is still rotating, so no, you don't subtract the rotational energy. --Trovatore (talk) 20:38, 30 December 2012 (UTC)[reply]
Per proton, "The rest mass of the quarks are thought to contribute only about 1% of the proton's mass. The remainder of the proton mass is due to the kinetic energy of the quarks and to the energy of the gluon fields that bind the quarks together." Interesting! 86.171.174.84 (talk) 23:44, 22 December 2012 (UTC)[reply]
From what I've learned rest mass usually describes a single particle. When there is interaction between particles, invariant mass is used instead since they are no longer all "at rest". Zhieaanm (talk) 02:05, 23 December 2012 (UTC)[reply]
Then why at the beginning of article, they describe rest mass and invariant mass are the same thing?184.97.227.164 (talk) 21:01, 23 December 2012 (UTC)[reply]
Perhaps because they are when one is talking about a single particle (but only then)? If so, this equivalence needs to be qualified. Thus, I presume that one should talk of the invariant mass of a spaceship rather than its rest mass. The description of the mass of a proton above only makes sense with this interpretation: The total rest mass of the particles constituting a proton and the invariant mass of the proton are very different. — Quondum 10:55, 27 December 2012 (UTC)[reply]

December 23

Columbia River Dams

If the Columbia River Dams had not been built what would be the effect on the last 80 years economy? Did the Columbia River Dams have any long term effect on the financial and political strength of United States?

Guy Swanson. Spokane. — Preceding unsigned comment added by 67.185.227.135 (talk) 02:19, 23 December 2012 (UTC)[reply]

They certainly had a major effect, but evaluating what things would be like without them is a difficult task, because the money not spent on them might have gone to some other major project which would have been just as beneficial (see opportunity cost). StuRat (talk) 06:47, 23 December 2012 (UTC)[reply]
The area would have been deprived of hydropower and would have had to resort to power from fossil fuels and nuclear fuel. But the Native Americans would have been able to continue fishing at Celilo Falls. ←Baseball Bugs What's up, Doc? carrots07:31, 24 December 2012 (UTC)[reply]

Scientific racism and pseudo science?

I came across this video on YouTube, link, and it basically involved two YouTubers ranting about whether race exists or not.

The point I have a question about starts at 12:12 and ends at 14:53 where the YouTuber in question, fringeelements, was trying to refute the claim that Eurasians have more in common with Africans than Africans do with each other which cited this paper as evidence for this claim. He contended that this study only analyzes SNPs and not entire genomes, and then cited this paper as evidence that the possibility that individuals of one self identified race are more genetically similar to individuals of another self identified race than they are to individuals of the same self identified race are virtually zero. He then goes on to point out that the opponent he was arguing against invoked Lewontin's Fallacy.

I'm not entirely sure if what he's saying is true or not. Are there any problems with his argument? Is it true that Eurasians are more genetically similar to Africans than Africans are to each other? Malamockq (talk) 04:07, 23 December 2012 (UTC)[reply]

What's an African? HiLo48 (talk) 04:36, 23 December 2012 (UTC)[reply]
LOL, African people. Bonkers The Clown (Nonsensical Babble) 04:45, 23 December 2012 (UTC)[reply]
  • "Similarity" is vague and undefined. The evidence points to all extra-African human populations having evolved from one subbranch of humanity, with all the other branches being African. (This is a simplification, but clear enough.) You'll get a lot of ideological squealing over this, as the Lewontin Fallacy article shows. The out of Africa theory article addresses the main points. μηδείς (talk) 04:49, 23 December 2012 (UTC)[reply]
Races aren't black and white (no pun intended), a race is defined by a continuous spectrum of genetic profiles with arbitrary boundries. For instance, a person of equatorial-eastern African descent would be expected to be tall, dark skin colour, and a particular skull shape, hence facial features; However, a short person with that descent can not be excluded. The point I'm trying to make is that a race is not discrete, it is an heuristic term used to describe a common combination of genetic ranges, and very often containg large overlaps. Plasmic Physics (talk) 05:19, 23 December 2012 (UTC)[reply]
You could say that. Or you could say people are all one species, and interbreed. μηδείς (talk) 07:54, 23 December 2012 (UTC)[reply]
They aren't mutually exclusive. There's no logic-reason why you can't say both. Plasmic Physics (talk) 10:17, 23 December 2012 (UTC)[reply]
You have to realize also that the differences amongst different people in Africa are greater than those in the rest of the world put together so lumping all Africans together is really rather silly. Saying "Eurasians have more in common with Africans than Africans do with each other" has rather a lot of things wrong with it. You could say for instance instead that Eurasians have more in common with one group of people in Nigeria than that group does with the two other main groups in Nigeria. Dmcq (talk) 00:52, 24 December 2012 (UTC)[reply]

oranamental conifer

Which one of the below is an ornamental conifer? pinus cedrus araucaria cycas sequoia — Preceding unsigned comment added by 117.201.244.229 (talk) 05:30, 23 December 2012 (UTC)[reply]

See the following:
StuRat (talk) 06:42, 23 December 2012 (UTC)[reply]

Do your own homework. Whoop whoop pull up Bitching Betty | Averted crashes 15:08, 23 December 2012 (UTC)[reply]

Why is the charge of quarks exactly proportional to the charge of electron?

For example, why is charge of up quark exactly equal to two thirds of the charge of an electron and down quark one third? How is it made to be like that? Or are the charges not properties of the quarks themselves but a result of interactions between those quarks?--Inspector (talk) 07:26, 23 December 2012 (UTC)[reply]

If we accept the numbers of up and down that combine to make a proton and a neutron and that those combinations have charges or +1 and 0 respectively, it's basic algebra to determine the charges of up and down themselves. The charge is an actual intrinsic property of the quarks themselves. We have plenty of articles with titles such as quark, up quark, etc. that have details of the history of their discovery and individual properties. DMacks (talk) 08:04, 23 December 2012 (UTC)[reply]
I have looked at the articles and the question isn't answered. Quarks have the charges they do. Electrons have the charges they do. But there is no explanation given as to whether this elegant ratio derives from some deeper principle of quantum mechanics, or if that's simply what's measured and there is no apparent need for it to be that way. Someguy1221 (talk) 08:06, 23 December 2012 (UTC)[reply]
If it were not so, then uncharged atoms could not exist, and chemistry would be very different, but that isn't an explanation. There must be something more fundamental about the nature of charge that we have not yet discovered (or is not yet explained in Wikipedia articles). Dbfirs 08:20, 23 December 2012 (UTC)[reply]

The charges of leptons and quarks are determined by underlying symmetries of the standard model. In particular, they lead to the following Gell-Mann–Nishijima like formula for the electrical charge of quarks (q) and leptons (l):

where the third component of the weak isospin is , the weak hypechrage of leptons (electron and neutrino) is (by definition) , so that electron has an electrical charge of and neutrino of . The sum of electrical charges of leptons is therefore . For quarks the same sum is , taking into account that quarks come in three colors.

Another important consideration is cancellation of the chiral anomaly, which requires that . The result is that . The final result is that quarks have electrical charges of and . Ruslik_Zero 12:49, 23 December 2012 (UTC)[reply]

A propos of nothing, for some reason I find your typo hyperchrage to be very appealing. If I were ever to get into some MMORPG, I think I might use that as my avatar name. --Trovatore (talk) 21:57, 23 December 2012 (UTC) [reply]
It's not a typo, we even have an article on weak hypercharge. Nonetheless, it is appealing and would make a cool screen name.Phoenixia1177 (talk) 17:32, 25 December 2012 (UTC)[reply]
It is probably better to think of electric charge as a kind of twist in space rooted at the electron or quark rather than as a property of the particles themselves. As a geometric property of space it is quantised and it doesn't matter what you do to the quark or electron itself - it remains the same even if you speed up an electron near the speed of light for instance. Dmcq (talk) 10:45, 24 December 2012 (UTC)[reply]

Instinct and sex positions

Suppose a young human male and female want to copulate, but they have absolutely no information how to do it (no experience, no porn, no friends stories, no Discovery channel etc). Which sex position they choose instinctively?

I don't need an advise, lol. It's about biological vs cultural in human beings. --Ewigekrieg (talk) 09:15, 23 December 2012 (UTC)[reply]

I'd say something face-to-face, based on sociological norms. That being said, that's my 2012 American sociology talking. DRosenbach (Talk | Contribs) 20:08, 23 December 2012 (UTC)[reply]
Is there any way of knowing? Would all such couples instinctively go for the same position? If they'd watched dogs and other animals, it'd probably be something other than the missionary position. -- Jack of Oz [Talk] 22:20, 23 December 2012 (UTC)[reply]
I figured that no information [on] how to do it (no experience...no Discovery channel, etc.) would exclude the sort of thing include. DRosenbach (Talk | Contribs) 23:02, 23 December 2012 (UTC)[reply]
If they've had no exposure of any kind to sexuality, why would it even occur to them? ←Baseball Bugs What's up, Doc? carrots23:22, 23 December 2012 (UTC)[reply]
You're under the influence of Western religion's oppressive view of sexuality as something shameful that spoils "innocent" minds, and that must be hidden away at all costs to prevent the spread of happiness. Bacteria know how to have sex. Domestic dogs that have never seen other dogs know how to have sex. Solitary animals like the tiger don't travel in packs and rarely interact with members of their own species, yet they have no problem having sex. Do you suppose all these organisms got their information from Discovery Channel?
EDIT: Scray pointed out that I never directly answered the question of "why would it even occur to them?" I meant to suggest that the human sex drive is instinctive. As other editors pointed out, sex is fundamental to the species' survival. It is also extremely old--sexual reproduction evolved 3 billion years ago, which is about a million times longer than the entirety of human history. I don't know the mechanism by which a human's hormones cause him/her to have sex, but I'm saying that such a mechanism is deeply ingrained in every animal that reproduces sexually, because reproduction is literally the only goal that evolution can be said to have.

--140.180.249.194 (talk) 01:10, 24 December 2012 (UTC)[reply]

Of course it would occur. Wow, what is this delightful person in front of me? I want to get closer. I want to hug. Still need more closeness. What a nice recess for my growing erection. I will take off my loincloth if it's too uncapacious to comfortably allow this. (as appropriate): I will take off my penis gourd because I feel so little of you there and so much everywhere else. (inevitably: I want to see you naked. Wow this must be even intimater. Let's try it. If you don't call intracrural a position.) Sagittarian Milky Way (talk) 00:40, 24 December 2012 (UTC)[reply]
I'm guessing they would figure it out. Sex is as built-in a function as we have, after perhaps breast feeding and fear of falling. Besides, the shapes are pretty self-explanatory to any curious mind.
My totally unreferenced guess as to the original question: Standing, face-to-face. --Trovatore (talk) 23:31, 23 December 2012 (UTC)[reply]
See this article. The bonobo, one of the closest human relatives, uses a variety of sex positions. The most common is doggy style, followed by missionary position. Bonobos also engage in tongue-kissing and oral sex. Just in case you still think human sexual instincts are learned from Discovery Channel, bonobos also engage in clitoris rubbing (amongst females), scrotum rubbing, penis fencing, masturbation, and mutual masturbation between an older male and a younger male. --140.180.249.194 (talk) 01:10, 24 December 2012 (UTC)[reply]
I really have no idea what this debate is, but I am happy that you are entering the level of bonobo awareness that results in pedantically preaching to everyone your knowledge of bonobo practices to the point of defeating your own points. The reason why you—and others—are aware of bonobo's behavior is exactly because of the world of the media educating you on exactly one theory on bonobo behavior. If you do some more research, you would realize that bonobo's are not some magical sex monkeys & folks are not as convinced about their behavior as you seem to imply. --SpyMagician (talk) 01:14, 24 December 2012 (UTC)[reply]
I am answering the OP's question by assuming that humans and bonobos have the same "natural" sex positions. Considering that bonobos are a close human relative, and that everyone else has been guessing without any references whatsoever, I think that's a pretty good response. If you want to link to a specific reference supporting your claims, please do so, because the bonobo article supports almost every single one of my claims. --140.180.249.194 (talk) 03:29, 24 December 2012 (UTC)[reply]
No and a someone up there was telling something about discovery channel hahaha
man... REALLY!?
I guess it would be just like all the other mammals, it’s true that I’m influenced by… “Discovery channel” but at least is the first position that come to my mind


Have a question about Avogero's law.. Can anybody solve it?

Avogedro's law says that every gas's gram molecular mass has 6.02X10^23 atoms and its volume is 22.4 litter; its also applicable for solids and liquids. But as we know 1litter water/steam=1kg/1000g water/steam... So how can 18g(gram molecular mass of water) water/steam be 22.4 litter??? — Preceding unsigned comment added by Rishader (talkcontribs) 11:17, 23 December 2012 (UTC)[reply]

Who said that it is also applicable for solids and liquids? ~22.4 is also only correct for an ideal gas. Plasmic Physics (talk) 12:13, 23 December 2012 (UTC)[reply]
One gram of a gas (or any other substance) does not contain 6.02 x 1023 atoms, but approximately that number of (protons + neutrons). The weight of 6.02 x 1023 water molecules is (6.02 x 1023 u/g) x 18 u = +/- 18 g, because the weight of one water molecule is about 18 u. Avogadro's law, which implies that 6.02 x 1023 molecules occupy 22.4 liters is only applicable to gasses. However, Avogadro constant can be used in converting weight units of any substance, gasses, liquids and solids alike. - Lindert (talk) 13:40, 23 December 2012 (UTC)[reply]
Thanks everyone. Lindert properly solved my problem. Thanks to u specially.. I wasn't clear about the Avogadro Number uses. I mixed up with the Law — Preceding unsigned comment added by Rishader (talkcontribs) 16:11, 23 December 2012 (UTC)[reply]

I don't understand anything said above, but one Mole of a molecular substance contains 6.02X10^23 molecules of it, whether they be O2 or NH3. One can figure out their weight in grams by summing the molecular weights times the number of each atom in the formula. This is rather simple chemistry. And the name is Avogardo Avogadro, BTW.μηδείς (talk) 21:20, 23 December 2012 (UTC)[reply]

You're maybe thinking of the Italian judge (advocato) or the fruit (avocado). This one is Avogadro. -- Jack of Oz [Talk] 21:30, 23 December 2012 (UTC)[reply]
Just by the way, Italian for "lawyer" is avvocato, with the double v and no d. And avocado is a vegetable. --Trovatore (talk) 21:50, 23 December 2012 (UTC) [reply]
I was continuing the time honoured tradition of making deliberate errors while correcting others for their making of deliberate errors while correcting others .... And I see you're also getting into the swing of things with your vegetable claim, contrary to what our article says. Who's next? :) -- Jack of Oz [Talk] 22:16, 23 December 2012 (UTC) [reply]
Our article is wrong. Well, it's right, from a botanical point of view, which I suppose is what it's getting at. But from a culinary point of view it's flat wrong to call avocado a fruit, as it is neither sweet nor sour. --Trovatore (talk) 23:04, 23 December 2012 (UTC) [reply]
"Knowledge is knowing a tomato is a fruit. Wisdom is knowing you don't put tomatoes in a fruit salad." -Unknown. The same sentiment works for avocados. And cucumbers. And squash. And chili peppers. And... --Jayron32 23:43, 23 December 2012 (UTC)[reply]
The correct solution is to realize that the culinary usage of the word fruit is distinct from the botanical sense, and culinarily, none of those things is fruit. In fact, they are all completely unambiguously non-fruit. Rhubarb, on the other hand, is a culinary fruit but not a botanical one. --Trovatore (talk) 00:01, 24 December 2012 (UTC)[reply]
So, what you're saying is this: There's more than one valid way of classifying edible things, and according to one of those ways, the avocado is a fruit. I chose that way. I never disputed that there are other ways. I just don't get what you're going on about. Must be the silly season. -- Jack of Oz [Talk] 00:19, 24 December 2012 (UTC) [reply]
Hehe, was just a misfortunate unspeling, as you can see from the redlink. μηδείς (talk) 21:32, 23 December 2012 (UTC)[reply]
Worth an {{R from misspelling}}, perhaps? Tevildo (talk) 21:34, 23 December 2012 (UTC)[reply]
In simple terms, Avogadro's law states that all gases occupy same volume.
Also note that one gram-molecular weight of any gas/compound would contain one mole (6.02 x 1023 number) of molecules.
···Vanischenu「m/Talk」 22:01, 25 December 2012 (UTC)[reply]

boltzmann brain

Are we humans considered as Boltzmann brain?203.112.82.129 (talk) 15:29, 23 December 2012 (UTC)[reply]

We are definitely a product of biological evolution. So, we are not a Boltzmann brain. --Ewigekrieg (talk) 15:46, 23 December 2012 (UTC)[reply]
It depends on how you look at it. On a macro scale, the brain is highly structured, with well defined functional areas and communication channels. Clearly that's an evolved situation. However, at a micro level, much of the brain appears to be randomly connected, and the detail nueronal wiring is not inherited. No two brains are identically wired, even in newborn indetical twins. Further, computer simulations of randomly connected large numbers of "nuerons" shows that quite realistic looking brain waves, that show a vigorous response to sudden stimuli and then gradually settle down to a well defined rhythm, just like the real brain does, generally appear in ramdonmly connected nets. However, the processing power of randomly connected nets in such computer simulations is often very resource inefficient, at least as far as simulations I have done go. One would expect natural selection to eliminate anything that is wasteful.
Our brains are, as far as nueronal interconnection goes, partly nature and partly nuture. Many nuerons start out randonly connected, but as we learn, skills increase by selectively dropping connections (and even entire nuerons) and adding others. Does something that satrts out randomly connected, and ends up after years of use efficiently structured mean that it is no longer a Boltzman network?
It seems to me though, that looking at the brain as a Boltzman structure is not terribly useful.
Wickwack 120.145.48.199 (talk) 13:56, 24 December 2012 (UTC)[reply]
I think you don't understand the Boltzmann brain argument, the whole point of which is that our brains are not Boltzmann brains. More precisely, the argument is that some cosmological scenarios predict that more intelligent life should arise through random fluctuation than through evolution (because there's so much vacuum for quantum fluctuations to happen in), and those scenarios should be ruled out on that basis. -- BenRG (talk) 17:18, 24 December 2012 (UTC)[reply]
The Wiki article on Boltzman Brains begins with the sentence A Boltzmann brain is a hypothesized self-aware entity which arises due to random fluctuations out of a state of chaos - which can be interpreted as function arising out of chaos - just as the human brain appears to work at the micro level - and does not at all show that show that something arising out of a random beginning should be ruled out. Perhaps the article should be rewritten if it does not give the view that you have presented. But I think not, for the reasons I gave earlier. If the Boltzman Brain paradox is due to not acepting that intelligence can come from random noise, then that is like religion - it is an aspect of faith that our brains are not Boltzman brains. Wickwack 60.230.194.179 (talk) 03:00, 25 December 2012 (UTC)[reply]
Hi guys. Wickwack, no, nonacceptance of the hypothesis is not like religion. Without sufficiently valid physics in place its a spherical cow, thus I'm skeptical that Boltzman brains are even possible, let alone likely. -Modocc (talk) 04:58, 25 December 2012 (UTC)[reply]
Yes, the article should be improved, like most of the physics-related articles on Wikipedia, which are surprisingly bad in general. There's nothing wrong with talking about the role of chaos in the human brain, but "Boltzmann brain" has a specific meaning and it's not that. See here for more information. -- BenRG (talk) 19:07, 26 December 2012 (UTC)[reply]

Bromeliad species? Prognosis?

I usually get my sister an Amaryllis for Christmas, but this year I bought a bromeliad that is of this species. Can anyone identify it? Are the plants semelparous? The one I got her is about a cubic foot in spread (including root mass) and has three spikes on two shoots. Should she just enjoy watching it bloom as it dies, or can she cut off the blooms and expect further growth and future blooming? Thanks. μηδείς (talk) 21:13, 23 December 2012 (UTC)[reply]

This plant looks like a vriesia which has a number of varieties. yours looks like 'Drako' or Miranda'. After flowering they usually throw offshoots which provide the next generation of flowers. These can be left on the parent plant or repotted to make another plant. Richard Avery (talk) 07:57, 24 December 2012 (UTC)[reply]
Actually, from your link, I am sure it is Bromeliad Vriesea Charlotte - Tawi. Happy to learn this plant will live long if taken care of. Maybe I will keep it for myself, since my sister kills everything else. μηδείς (talk) 20:59, 25 December 2012 (UTC)[reply]

December 24

Balangoda_Man is homo erectus or Homo Sapien

My Question is related to the article in http://en.wikipedia.org/wiki/Balangoda_Man.

According to the article, balangoda man is homo-erectus. But it is dated 37,000 yrs ago. According to graph and the article connected to http://en.wikipedia.org/wiki/File:Humanevolutionchart.png homo erectus were extinct about 500,000 years ago.

These two are contradicting each other. Is balangoda man really a homo erectus. If yes, why wikipedia still say homo erectus extinct 500,000 years ago.

Hope I can get a good answer with new findings.

Thank you. -Achala — Preceding unsigned comment added by 69.132.64.162 (talk) 04:35, 24 December 2012 (UTC)[reply]

My reading is that it says Homo Erectus was there before Balangoda man. StuRat (talk) 04:45, 24 December 2012 (UTC)[reply]
I agree with StuRat. The article discusses other hominids found in the same place as Balongoda Man. The article, in the first sentence, makes it clear that Balongoda Man is a Homo Sapiens (anatomically modern humans are always Homo Sapiens). The discussion of prior hominids is there to provide context for earlier human and hominid settlement in the area where Balongoda Man was found. --Jayron32 04:50, 24 December 2012 (UTC)[reply]

"alkaline gargle"

I remember watching a old film from the 1920's about cruise ship illness that said to treat tonsillitis use a "alkaline gargle" every 30 minutes. What exactly is a "alkaline gargle"?, Also lets say its Sodium Bicarbonate or similar I read of Sodium Bicarbonate causing chemical burns in the mouth (http://www.casereports.in/articles/2/1/Sodium-Bicarbonate-mouth-rinse.html) and would be surprised they would use it every 30 minutes for days without harm.--Wrk678 (talk) 08:16, 24 December 2012 (UTC)[reply]

Probably a salt water gargle, probably 1 teaspoon salt to 1 pint water, gargle and spit rather than swallow. --TammyMoet (talk) 09:05, 24 December 2012 (UTC)[reply]

I dont think salt is alkaline--Wrk678 (talk) 13:38, 24 December 2012 (UTC)[reply]

Indeed! An alkaline gargle is composed of water plus baking soda or potassium carbonate. Our article on baking soda lists several home uses, but doesn't seem to mention gargles. A quick Googling brings up all manner of odd things folks gargle with, of which baking soda is far from the most dangerous. As always, it's the dose makes the poison. Matt Deres (talk) 19:46, 24 December 2012 (UTC)[reply]

Wouldn't gargling with it every 30 minutes cause a chemical burn like this http://www.casereports.in/articles/2/1/Sodium-Bicarbonate-mouth-rinse.html Also unlike sodium bicarbonate I thought Potassium carbonate is quite caustic to skin (PH 11.5) and wouldn't work for gargling.--Wrk678 (talk) 20:50, 24 December 2012 (UTC)[reply]

That would depend on the ratio of chemical to water. I threw my last link in there more as a gag, but it turns out that that's actually the important one for you to read. In the wrong quantities, anything can be dangerous to consume. In the right quantities, anything can be made safe. For example, botulinum toxin is among the most toxic substances on earth, yet it's routinely used as "Botox" treatments for reducing the signs of aging. Putting pure potassium carbonate into your mouth is a very different proposition to putting a small amount into a glass of water and gargling. Matt Deres (talk) 21:30, 24 December 2012 (UTC)[reply]


Yes, but when you are done gargling and the water evaporates the baking soda or potassium carbonate would dry on your mouth/throat and be significantly concentrated. --Wrk678 (talk) 18:35, 25 December 2012 (UTC)[reply]

calculate ph change that takeplace when 100ml of .05m naoh and .05m hcl are added to 400ml of buffer soln that is .2m in nh3 and .3m in nh4cl

calculate ph change that takesplace when 100ml of .05m naoh and .05m hcl are added to 400ml of buffer solution that is .2m in nh3 and .3m in nh4cl — Preceding unsigned comment added by 182.185.115.249 (talk) 08:50, 24 December 2012 (UTC)[reply]

Why don't you try your own homework, and let us know if you get stuck! Someguy1221 (talk) 09:38, 24 December 2012 (UTC)[reply]

Suitable conditions for ventilating a residential property to reduce humidity

Can someone help me by determining a formula that I can use to calculate suitable times to ventilate a house that gets quite damp. I'm thinking that I can't simply rely on hygrometer readings because the air temperature also affects the capacity of air for water. I want to make sure I ventilate on days that the external air has a lower water content than the internal air which presumably is more effective than relying on a dehumidifier (which we also have on constantly, removing about four litres per day). — Preceding unsigned comment added by 92.28.74.239 (talk) 14:58, 24 December 2012 (UTC)[reply]

As the weather changes from day to day, for most regions of the World, you will do better to install sensors that sense 1) the inside temperature and relative humidity, and 2) the outside temperature and humidity. The position of the sensor needto be carefully chosen and shielded from sun, reflected heat, etc. For each temperature and relative humidity, the absolute humidity (air moisture load) can be calculated from a psychrometric chart or standard formula. This can easily be programmed into a micro-controller, indeed such programmed controllers are commercially available. Run the ventilation fans whenever the outside air moisture load is less than the inside load.
Depending on the area and on the building, you may find this ineffective, as moisture will difuse back in during no ventilaton periods. It takes only a little infiltration thru cracks, door gaps, and the like, to upset this.
You will get better answers if you sign off posts with a pen-name that you consistently use. Wikipedia is a free world - you don't have to. But you will get better answers if you do. If we recognise the name, we might even do a bit of research and find some good links.
Wickwack 58.169.242.150 (talk) 16:00, 24 December 2012 (UTC)[reply]
What you should look at is the dew point. Most good weather sites will list the dew point along with the temperature and relative humidity. If you tell me where you live, I'll try to find a site which lists this info for your area. A dew point below 60°F is comfortable, so you could ventilate on those days. However, I wouldn't say you should ventilate whenever the exterior dew point is lower than the interior. For example, if the external dew point is 80°F and the internal dew point is 81°F, it's going to make very little difference. You'd do better to use dehumidifiers on such days.
Also, you can tell when it's humid out without instruments. Does it feel "sticky" ? Then it's too humid.
Finally, try to eliminate sources of humidity inside the home. If you take a shower, close off the bathroom from the rest of the house and use the exhaust fan, if you have one, or open the window, otherwise. Avoid cooking which generates lots of steam. Keep all pots and pans covered, and use an exhaust fan, if you have one. StuRat (talk) 19:28, 24 December 2012 (UTC)[reply]
Stu, the OP used the words "the house gets quite damp" - this suggests he is concerned not about human comfort but about condensation/damp surfaces. This can be a problem, causing deterioration of household items even when the temperature is low enough that humans feel comfortable even at 100% relative humidity. Further, although dew point is essentially the same thing as I described above (the air moisture load) it is gnerally inadvisable to go on the dew point as published by weather authorities, as the dew point varies locally quite a bit. For example, at the time of me writing this, our city weather authority, which measures within a City park and at the airport (both clear sites away from water and buildings) has just updated on their website the data for our area as follows: Temperature 34.2 C, Rel Humidity 29%, Dew Point 14.1 C. The actual data measured on my instruments at my house, which happens to be adjacent to a large river and is surrounded by large trees, is Temp 29.9 C, Rel Humdity 29% - this gives a calculated dew point of 10 C. Dew point sensors are not generally commercially avaliable, but temperature and relative humidity sensors are - that's why I did not mention dew point in my 1st post.
I agree with your advice about eliminating sources of humidity though. It may also be that the OP's real problem is rising damp in the building structure, seeing as we have no idea of his situation. Wickwack 60.230.194.179 (talk) 02:45, 25 December 2012 (UTC)[reply]
There might be a language difference on "damp". Here, we might say "the air is damp" meaning the humidity is high. I don't think there's any indoor temperature where 100% relative humidity is comfortable (not at normal air pressure, anyway). StuRat (talk) 21:56, 26 December 2012 (UTC)[reply]
I'm quite happy with the language point - there are a multitude of examples of where an Australian (eg me), and Englishman, and an American (eg yourself, Stu) use the same English language words in a different way. However, yes, you can feel comfortable in 100% rel humidity. The body gets rid of heat in 3 main ways: radiation, convection, and sweating. 100% rel humidity will render sweating useless, but if the air temperature is low enough, the first 2 methods still work. We sometimes get 100% rel humidity where I live, quite often at night. I personally feel quite ok at 100% rel humidity if the temperature is below 25 C, althoough if I am physically working hard, I may need it as low as around 18 C. It depends on what you are used to as well. I have a British textbook on airconditioning that states that noticeable sweating is an emergency reaction and may cause some distress. Sweating noticeably is what we do normally in Australia. Wickwack 58.164.228.63 (talk) 01:39, 27 December 2012 (UTC)[reply]
Well, you're unusual then. I can provide many sources saying that 100% humidity is outside the comfort range. Here's a few: Relative_humidity#Comfort, [3], [4]. Can you provide any saying it's comfortable, even for Aussies ? There are also a number of devices which have instructions saying the humidity should be limited to 95% (to avoid any possibility of condensation). The lacquer on my wooden floors also becomes tacky during extended periods of 100% humidity. StuRat (talk) 01:45, 27 December 2012 (UTC)[reply]
Your cited links don't actually say that humans cannot be comfortable at 100% rel hum. I can't actually think of any online references I can give. It is a fact that humdity can be 100% in certain countries, including some parts of Australia - but we live there anyway - you get used to it. I have several aircon textbooks and none would agree with me, but there is a simple reason for that - they all come from the USA and Britain. Its a bit like what they say about temperature - the British book says temperatures over 25 C may cause distress in some people, and 45 C will kill. There's places in Australia and Soudi Arabia where folk routinely work in 45 C heat (I was one of them, but take me to Alaska and I might die of cold. But folk live there too). I haven't heard of floor treatment problems, but we do have problems with mold. Newspapers and other things printed on newsprint paper tends to accumulate black spots. Things made from cadmium plated steel can rust very quickly. Wickwack 58.164.228.63 (talk) 02:28, 27 December 2012 (UTC)[reply]
"Relative humidity above 60% feels uncomfortable wet. Human comfort requires the relative humidity to be in the range 25 - 60% RH." - That seems to quite clearly say that humans can't be comfortable outside that range. If Australians disagreed, I'd expect them to have set their own standards. I think it's just you who disagrees. I can see people from different regions varying their preferences some 10% or so, but not from 60% to 100%. Also, if people lived in an area for tens of thousands of years, then they might have physical adaptations to that humidity, but not after a few hundred years. I'm also wondering if "comfortable" has a different meaning to you, like "not cause death", versus what it means to the rest of us, being the preferred humidity. StuRat (talk) 03:19, 27 December 2012 (UTC)[reply]
Certainly, "comfortable" may mean a different thing to an Australian, than it does to (say) an Eskimo. I said as much when I talked about sweating. The British very much don't like being wet with sweat. To us it's normal - you can't aviod it when working. But a point that I have been making is that humans get used to the prevaling environment. It's not a matter of evolutionary adaption, it is a matter of acclimatisation. Last week in Western Australia, there was a catastrophic example, reported widely in local and British media. A teenage son came out from Scotland (a cold country in the middle of winter) to visit his father, who has been working in Australia (a hot country now in summer) for some time. The father took the son for a long walk in approx 47 C heat. The son went into heat stress and collapsed, emergency services were called, but the son died shortly after they arrived. The father apparently had no problems. I was born in Australia of European parents. When cousins and aunts have visited, they came, against our advice, in summer, because that's when the airfares are cheap. All they wanted to do is sit around in the house and not go out and see anything, because what seems pleasantly warm to us is unbearably hot to someone who normally has to cope with snow. Immigrants usually find that by their second summer, they've largely become used to it too. It matters not a whit what European and American textbooks (and wki articles) say - that's how it is. Setting standards has nothing to do with it - you can't change the climate with a pen. Fortunately, while we get very high humidity and high temperatures, we don't generally get both at the same time, or we would be in trouble. Wickwack 60.230.192.16 (talk) 04:16, 27 December 2012 (UTC)[reply]

Flame is matter or not

Is flame matter or merely energy ? I am confused because of two observable properties of flame: first, if we blow a flame it moves in the direction of wind (I think it is a property of matter) and second, we cannot capture or divide flame in two parts, but we can do this with matter. Sunny Singh (DAV) (talk) 15:33, 24 December 2012 (UTC)[reply]

Flame is matter. When something is burnt, various gasses and particles are produced that make up the flame. It is readily possible with the right apparatus to captue these gasses and separate them out. Also, by analysing the emitted light, the individual gasses can be identified. The red and reddish/yellow colour seen in many flames is carbon particles heated so that they are red or red/yellow hot, ie.e, are heated sufficiently to provide visible black body radiation. Various gasses give various colours. Wickwack 58.169.242.150 (talk) 15:47, 24 December 2012 (UTC)[reply]
That's a bit like saying dancing is matter, because we can confuse dancing with the body of the dancer if we do not pay attention to the semantics. Flame is probably better thought of as a manifestation of a process than as matter. — Quondum 06:31, 25 December 2012 (UTC)[reply]
I tend to agree with this view, though honestly it's an old and complicated question. I think it best put that fire is a state of transition, a self-catalyzing chemical reaction that happens to involve matter but is not defined exclusively by the mass involved but also by the transformative principles resulting from the interactions between the constituents of said matter and their innate thermodynamic properties. Now that's a far cry from a simple intuitive way of stating things, but at the same time I don't think it gets much simpler for this particular phenomena; as common as it may be in our daily lives, fire is simply not all that easy to intuitively comprehend. Snow (talk) 08:21, 25 December 2012 (UTC)[reply]
From what I've learnt: A matter is something that (need not have all the criteria):
  • Occupies space. Fire does not.
  • Has mass. Fire does not.
  • Can change state. Fire does not.
So it is not. Hope this helps and cheers. Bonkers The Clown (Nonsensical Babble) 14:48, 26 December 2012 (UTC)[reply]
The OP asked is flame matter. Taking Bonker's criteria:-
Occupies space: Yes it does. Flames comprise heated gasses and particles and most definitely occupy space;
Has mass: Yes, the heated gasses and particles certainly have mass;
Can change state: Flames are already in the gasseous and particle state; particles entrained in flames may comprise solids (eg carbon) and liquids (e.g., partly pyrolysed and unpyrolysed liquid fuels such as hydrocarbons) that do change state by evaporation within the flame.
So, yes, flames are most certainly matter - I would have thought that very obvious. Bonkers is indeed Bonkers. Wickwack 58.164.226.231 (talk) 15:51, 26 December 2012 (UTC)[reply]
Oh yeah, yeah. I sure am Bonkers. Yes, fire is NOT matter, more of energy... As of flame, I would suppose its some form of plasma. Bonkers The Clown (Nonsensical Babble) 06:07, 27 December 2012 (UTC)[reply]

I read in "Encyclopædia Britannica 2009" that -
Matter can be defined as anything that has inertia and that experiences an attractive force when in a gravitational field. What is flame according to this definition ? Sunny Singh (DAV) (talk) 06:24, 27 December 2012 (UTC)[reply]

This definition would be more complex. The simple criteria I have listed above. It's actually arguable/negotiable. Flame could be listed as a semi-gaseous/plasma state of matter. Same debate is as to whether light is matter. Or not. It all boils down to the exact definition of matter. Bonkers The Clown (Nonsensical Babble) 06:31, 27 December 2012 (UTC)[reply]

Fire in absence of oxygen

Sun is the hot burning ball of fire. If this statement is correct, then my question is reasonable. Oxygen is a necessary condition for fire, but around the sun there is almost vacuum and sun is a ball of fire. Without oxygen how this fire is sustained. Sunny Singh (DAV) (talk) 15:55, 24 December 2012 (UTC)[reply]

It's NOT fire. The sun glows due to nuclear fusion. Whoop whoop pull up Bitching Betty | Averted crashes 16:05, 24 December 2012 (UTC)[reply]
The glow at the surface at the sun is purely thermal and has nothing directly to do with nuclear power. This is exactly the same general sort of glow that old Incandescent light bulbs have. They use electricity to generate heat and then glow because of the heat. The switch to LEDs moves us much closer to a direct connection from electricity to light. Old Sol will continue to glow as a White dwarf long after it has stopped fusioning. Hcobb (talk) 16:22, 24 December 2012 (UTC)[reply]
And where does that heat come from? Nuclear fusion. (BTW, the switch is to CFLs, not to LEDs.) Whoop whoop pull up Bitching Betty | Averted crashes 18:31, 24 December 2012 (UTC)[reply]
(ec)A link to Black-body radiation might help complete the link between nuclear fusion and the glow described above. in my home we've already replaced a few of our incandescent bulbs with LEDs - and a few CFLs with LEDs; LEDs do provide a tighter connection between electricity and light. -- Scray (talk) 20:01, 24 December 2012 (UTC)[reply]

Some small part of that heat is still from the gravitational formation of the sun. Hcobb (talk) 19:55, 24 December 2012 (UTC)[reply]

First of all, oxygen is not always needed for burning. You think it is because it's a very common element and very powerful oxidant. Fluorine can burn bricks and water without a spark--see this video.
As for the Sun, nuclear fusion occurs in its core, and the energy is carried as gamma photons across the radiation zone. This takes about 200,000 years. When it reaches the convection zone, the heat is carried by convection, meaning "it touches something hot, so it gets hot". The convection zone glows, as other people have said, due to blackbody radiation, just like how iron in a blacksmith's furnace glows. --140.180.249.194 (talk) 21:06, 24 December 2012 (UTC)[reply]
Actually, convection means "hot gases rise, cold gases fall" -- same reason why a hot-air balloon ascends even with a heavy load. "It touches something hot, so it gets hot" would be conduction. 24.23.196.85 (talk) 01:06, 25 December 2012 (UTC)[reply]
I think what he means is that the gamma photons are absorbed by the ions in the convection zone, which become hot and move outwards, towards the surface of the sun. This heats the photosphere at the surface, which glows as a result. Whoop whoop pull up Bitching Betty | Averted crashes 18:57, 25 December 2012 (UTC)[reply]
You just did. These things take time. Relax and enjoy the kudos. -- Jack of Oz [Talk] 07:04, 26 December 2012 (UTC)[reply]
I have never actually had one. Do they still even market them? μηδείς (talk) 01:36, 27 December 2012 (UTC)[reply]

Is the electric field a side effect of the Pauli exclusion principle?

Consider two electrons. We have two different reasons why these cannot be squeezed into the same overlapping state. The Electric field shows that it would take an infinite amount of energy to move two point particles each with a -1 electron charge into the exact same spot and the Pauli exclusion principle states that two electrons must differ by at least one quantum number.

So is electrical repulsion simply a distributed application of the exclusion principle? I.e. is it the exclusionary interaction of the fringes of the wave packets what causes like charges to repel each other? Hcobb (talk) 20:20, 24 December 2012 (UTC)[reply]

No. The electrical force is completely independent of the Pauli exclusion principle. Both behaviors happen to affect electrons, but otherwise they are not related. Dragons flight (talk) 22:05, 24 December 2012 (UTC)[reply]
To put the question another way, what changes would be needed to Exchange interaction to produce an effect that at the classical limit behaved like electromagnetism? Hcobb (talk) 04:29, 25 December 2012 (UTC)[reply]
Just to set something straight: The Pauli Exclusion Principle doesn't really say anything about charged point particles occupying the same point in space. The PEP operates on a completely different model of what an electron is. The PEP merely says that two fermions cannot have identical quantum states. The first thing about the quantum model of particles is that particles don't have a defined precise location, so it isn't meanigful, in the world where Pauli operates, to think of an electron as a little point of electric charge sitting around. You're describing two different models of what an electron is, and there isn't a lot of crossover between them. --Jayron32 04:55, 25 December 2012 (UTC)[reply]
And to possibly answer your question a little more, there is no way in which one could interpret the PEP such that it remotely behaves like an electric field. For example, two electrons repel each other electrically, regardless of spin (though magnetic effects do depend on whether they are parallel or antiparallel). The PEP changes completely depending on spin being parallel or antiparallel. A further point of interpretation: there is no field (and hence force) generated by the PEP. The only force associated with the PEP should be interpretated as momentum exchange carried by the electrons (i.e the wavefunction), not by an intermediary force field. One would never expect to be able to produce any kind of classical limit that agrees. — Quondum 05:15, 25 December 2012 (UTC)[reply]

The wrinkle that might make this work is applying exclusion to all the virtual electrons between the two real charged particles. It's just at the classical limit that the Dirac sea would produce the classical electromagnetic field. Close up things would behave in a more quantum manner. Hcobb (talk) 08:41, 26 December 2012 (UTC)[reply]

I fail to see how. What you are suggesting attempts to attribute charge and electrostatic force to the PEP, and thus the idea of the particles being charged in the first place cannot be used. And you would not expect to obtain electrical attraction between positrons and electrons, and would also not expect electrostatic interaction between particles of different types. Fermions of the same type, e.g. neutrons would be expected to repel each other. Besides which, you would have to find a wavefunction for the fermion field that produces the correct expectation value for energy–momentum transfer (i.e. the electromagnetic stress–energy tensor, giving positive, negative and shear pressure of the electromagnetic field). The long-range transfer of momentum requires quite a definite momentum density, which has to propagate at the speed of light; not great for massive transmitting force via their own momentum, virtual or otherwise. Magnetic fields seem to be utterly inexplicable through this mechanism. "might make this work" is being rather optimistic. — Quondum 18:22, 26 December 2012 (UTC)[reply]

Highest altitude for manned post-Apollo mission

What is the highest altitude reached in human spaceflight in the last 40 years? The closest I can find in List of spaceflight records is the 1374km altitude record (set in 1966) for a non-lunar mission. Dmytro (talk) 22:21, 24 December 2012 (UTC)[reply]

In case you didn't notice, this list specifically excludes lunar missions -- for obvious reasons, the "altitudes" reached in those missions were so high that Earth's gravity field was no longer dominant, therefore the whole concept of "altitude" defined as distance from Earth becomes kind of nebulous in these cases. FWiW 24.23.196.85 (talk) 00:59, 25 December 2012 (UTC)[reply]
I think the Hubble launch and service missions. Which one was highest I am not certain. Rmhermen (talk) 02:29, 25 December 2012 (UTC)[reply]
I looked at the Hubble Space Telescope shuttle missions, and the highest altitude appears to be around 620km in STS-82 (1997) as confirmed here. Most of that mission was at a lower altitude, but after servicing, Hubble Space Telescope was boosted to its highest altitude ever. Still, this is slightly less than one tenth of Earth radius and is less than 0.2% of Earth-Moon distance, and the lack of higher flights is one of the most remarkable regressions in the history of human exploration. Dmytro (talk) 04:17, 25 December 2012 (UTC)[reply]
One should perhaps not be too quick to use lack of manned missions as a criterion of regression in human exploration. The advances in unmanned exploration often make the same amount of effort go so much further in terms of exploration that only a romantic would define progress in terms of manned missions. Far more realistic criteria would be the annual investment in exploration, the amount of support that new exploration enjoys, and the extent of new knowledge gained from exploration. I'm not saying that there has not been regression by these criteria though. — Quondum 09:59, 25 December 2012 (UTC)[reply]

December 25

Acetone

I used Acetone in my car. The vapors are so strong that it has irritated my eyes. Is there a vapor suppressing foam I can buy. Do you know any retailers selling this? Is there anything else I can use. What else can I do. Thanks — Preceding unsigned comment added by 174.111.55.161 (talk) 02:45, 25 December 2012 (UTC)[reply]

Open doors and big fan for a day or two. --Jayron32 02:47, 25 December 2012 (UTC)[reply]
You didn't learn from when you had this problem last year? DMacks (talk) 05:55, 25 December 2012 (UTC)[reply]
Or are you saying you can still smell it 16 months later ? StuRat (talk) 06:13, 25 December 2012 (UTC)[reply]
Nothing new under the sun, huh? Any chance Keeeeeeith is baaack? μηδείς (talk) 20:55, 25 December 2012 (UTC)[reply]

surface tension

why is there a colouring change when soap is reacted with milk? — Preceding unsigned comment added by 117.196.135.236 (talk) 16:40, 25 December 2012 (UTC)[reply]

What color does it change into? 76.23.194.179 (talk) 17:16, 25 December 2012 (UTC)[reply]
Ordinarily, milk is coloured white because of the Tyndall effect. When soap is added, the colloidal particles are fully dispersed. Plasmic Physics (talk) 20:37, 25 December 2012 (UTC)[reply]

balloons and fire

direct contact of a blowed balloon with fire can burst that balloon but when that balloon filled with water show to the fire it does not burst . why ? — Preceding unsigned comment added by 117.196.135.236 (talk) 16:48, 25 December 2012 (UTC)[reply]

Gases expand greatly when heated. For most situations, gases are governed by the ideal gas law: pressure(P) * volume(V) = number of gas particles (N) * boltzmann constant(k) * temperature(T). Or given a certain pressure (gas in a balloon will expand until internal pressure equals atmospheric pressure), V = N*k*T/P. If you heat a gas by flame, it will expand so much (the temperature will increase rapidly) that the balloon will burst. Water does not expand that much when heated, in fact it has a very high heat capacity. You have to convert the water into gas (i.e. by heating at 100C) to burst the balloon. 76.23.194.179 (talk) 17:15, 25 December 2012 (UTC)[reply]
I think what's more relevant is the heat capacity of the water inside the balloon which prevents the skin of the balloon from so quickly reaching melting point. I have never seen a balloon expand first before exploding due to a flame, which is what you are implying, IP 76, would happen. μηδείς (talk) 20:54, 25 December 2012 (UTC)[reply]
Once, my high school chemistry teacher filled a balloon with hydrogen gas and set the balloon on fire. It did not burst until the balloon burned through, igniting the hydrogen. Whoop whoop pull up Bitching Betty | Averted crashes 07:22, 26 December 2012 (UTC)[reply]
Another example that illustrates the heat capacity explanation of μηδείς is a burning candle standing in cold water: you should be able to get it to burn down a little way below the water surface level protected by a thin-walled cylinder of unmelted wax. — Quondum 07:37, 26 December 2012 (UTC)[reply]
I agree with Medeis - another common demonstration is placing a paper cup over a candle or bunsen burner. The water absorbs heat from the paper quickly enough that the paper doesn't ignite. You can actually bring the water to a boil this way. This is clearly the same kind of effect you describe, and the expansion of the water or air has nothing to do with it. 209.131.76.183 (talk) 18:01, 26 December 2012 (UTC)[reply]
You can boil water over a camp fire in a plastic soda bottle. Vespine (talk) 22:05, 26 December 2012 (UTC)[reply]
In fact, with a little skill, you can even melt lead or tin in a paper crucible... 24.23.196.85 (talk) 01:11, 27 December 2012 (UTC)[reply]
Those are three interesting demonstrations none of which I have ever seen done. μηδείς (talk) 01:33, 27 December 2012 (UTC)[reply]

tests for reality

What are some philosophical and scientific tests to help verify that reality is real, i.e. what we perceive is not an illusion? I know a recent one mentioned in the news is some big particle physics experiment or something. 76.23.194.179 (talk) 17:19, 25 December 2012 (UTC)[reply]

There are no scientific tests to prove to yourself that you are real. It's one of those things you need to take on faith. Philosophically, it's a question almost as old as philosophy itself. Different philosophers have approached the question in many different ways. Plato had the Allegory of the Cave for example. Rene Descartes took the rather pragmatic "cogito ergo sum" as his approach. If you want the philosophical end of being and existence, you'll want to look into the well-mined field of Ontology which deals with the question in some detail. --Jayron32 17:53, 25 December 2012 (UTC)[reply]
Claiming that we are in a simulation can exclude all arguments "pro-reality." Everything that you perceive, feel or think is simply part of this simulation. It's like the belief in god, everything that point towards a materialistic universe without a god is simply the wish of god. OsmanRF34 (talk) 20:35, 25 December 2012 (UTC)[reply]
Email me next time you're in New York. I'll come slap you up the head. You can tell me whether it's real or not. (Hopefully you aren't upset by not really being slapped upside the head.) μηδείς (talk) 20:51, 25 December 2012 (UTC)[reply]
This is the classic Johnsonian argumentum ad lapidem - the slapping may _feel_ real, but how can the slappee know that it _is_ real? Tevildo (talk) 21:17, 25 December 2012 (UTC)[reply]
Ask yourself how a brain in the vat, receiving the same inputs as your brain, would perceive the world any differently. (Answer: it wouldn't.) Contrary to what Jayron says, you don't "take on faith" the assumption that reality is real; you admit that you don't know. That may be hard for people who usually make up what they don't know, but if you don't know, then you don't assume anything.
That said, people have tried to make various arguments about why it's unlikely that we're brains in vats. Most of them reduce to Occam's razor. If we're brains in vats, there must be an external reality in addition to the reality that we experience, with intelligent beings who have the motivation to put brains in vats. Since this scenario is far more complicated and has no more explanatory power, we assume we're experiencing reality. --140.180.249.194 (talk) 21:48, 25 December 2012 (UTC)[reply]
There is no difference between taking something on faith and admitting that you don't know. They are two perfect synonyms for the same concept. In either case, you still need to operate as though what you perceive is reality. Either you admit you don't know, but you behave as though it were real, or you accept on faith that it is real. It amounts to the same thing. --Jayron32 23:06, 25 December 2012 (UTC)[reply]
The solution is simple. Define what is real by setting some simple parameters and conditions. If something passes those tests, it's real for the purposes of the definition. If it doesn't, it's not.
Now, whether these supposedly real things are really real ... well, who gets to say? It's all subjective. There isn't any such thing as objective reality. If you don't believe me, please provide one counter-example. That's all it would take. Nobody's ever succeeded, and they're never going to, because whatever we know of anything in the universe is what our senses tell us. Those senses are essential for us to perceive or know anything, but they also act as a barrier between us and those things. We can never know those things directly, extra-sensorily. Those things include everything about ourselves that we assume to be real. Descartes might have had a point with his "cogito ergo sum", because it's hard for a non-existent entity to think. But how do we know we're thinking? How do we know a rock or a tree don't think? What is thought, anyway? Do we get to decide what thoughts we have, or are we victims of "some vast eternal plan"? Does the universe really exist? Who could ever provide definitive and incontrovertible answers to these questions? Nobody. Doesn't stop us asking the questions, though. We're left with what we can perceive, and we try and work with that. It seems to be enough for most people. -- Jack of Oz [Talk] 22:38, 25 December 2012 (UTC)[reply]
I don't know which "big particle physics experiment" you're referring to - but it doesn't matter what it said because it can't possibly prove the existence of "reality" for any individual because you can't know whether the report of it that you thought that you read is "real". It's only your senses that report that this is what it said - and you know that you can't trust those senses. Truly, the only thing that gives you even a hint of a solution is something vague like Occam's Razor. The simplest and most practical and useful thing you can do is to assume that your senses are telling you the truth - until such time as it's proven that they are not.
I'd also like to suggest that it quite literally doesn't matter. So long as it's all consistent and your actions produce results in this seeming reality that seem correct, what do you care if we're really just brains in jars or a part of some alien intelligence's computer game? You still need to do the same things to avoid unpleasant feelings of hunger or thirst - your drives are still "real" to you and need to be attended to.
Conversely - we might find some strong pointers showing that the universe might truly be a simulation running in some mega-computer in the "real" universe. I'm spent my career as both a game programmer and a simulation programmer. It seems quite striking to me that there are a number of things that we observe about our universe that make it seem to have properties very much like the restrictions that a simulation might have:
  1. Quantum theory. When you look at the very smallest things - their behavior is random and quantized. In a finite precision computer, that's exactly what you'd expect. There would be randomness due to things like roundoff error and because of that finite precision, you'd see quantised behavior in all sorts of very small system. Old flight simulators from the 1980's used to have a precision of 1/256th of a foot - if you lived in that simulation, you'd say that the laws of physics quantised all distances to a "plank length" of 1/256th of a foot and invent complicated 'laws of physics' to explain that.
  2. Relativity. It's very convenient for the simulation for there to be a finite speed of light and a universal speed limit. It constrains the amount of the universe that you'd have to simulate with great precision and covers up the effects of latency between computers simulating different parts of the universe - and it also prevents humans from spreading outwards beyond the region which is simulated at high fidelity into further reaches of the universe where the computer could do a more sketchy simulation and not have to track ever single fundamental particle.
  3. The Big Bang. That's a very convenient way to hide the finite nature of time. You can't have the simulation running forever - it has to have a fixed start time. It has to be impossible to see what happened before that time - and starting things off as a singularity makes a lot of sense. This also imposes a limit on the size of the observable universe - which, again, is handy if your computer has finite memory.
  4. Why there is so little matter in all of this vacuum. Building your simulated universe like that allows you to have a vast universe with much less computational effort than if it were mostly full of matter.
  5. Entropy. Highly organized systems take a lot of simulation time - allowing things to degenerate into low grade thermal 'noise' keeps the simulation within reasonable bounds. If the system became more organized as time passed, the simulation computer would start to run more and more slowly.
SteveBaker (talk) 04:01, 26 December 2012 (UTC)[reply]
Steve, none of this makes any sense. Quantum mechanics is not cheaper to simulate; it seems to be impossible to simulate a quantum system on a classical computer without exponential slowdown. Even accurately simulating a single proton is far beyond the ability of any supercomputer. The enormous cosmos we find ourselves in could be eliminated without any practical effect on our lives. Those vast reaches of space are not empty, but filled with CMBR photons, which are not quite randomly distributed but show a slight anisotropy, first seen with COBE, which has been faithfully preserved for billions of years. The universe was not made on a budget. -- BenRG (talk) 17:19, 26 December 2012 (UTC)[reply]
I'm not saying that quantum mechanics is simpler to simulate - I'm suggesting that it's an artifact of any simulation on a finite precision computer that you'll see quantisation and other bizarre behaviors if you look on a small enough scale. Of course simulating even a tiny piece of our universe (like one gram of water) is insanely far beyond what any technology we can imagine could to - but remember that the laws of physics out there in the "real" universe could (and almost certainly would) be very different from our own - perhaps there is no speed of light limitation? That would perhaps allow computers to be very much faster than they are in our universe and (conceivably) make simulating vast numbers of fundamental particles - even with quantum mechanics - a fairly simple business. I don't know (obviously) - but it seems to me that some of the stranger things that we find when we look hard enough could be revealing "bugs" or "artifacts" that might reveal the likelyhood that we're being simulated. SteveBaker (talk) 04:07, 27 December 2012 (UTC)[reply]
I think that makes a lot of sense, Steve. Actually the real universe could have supercomputers more vast in power than we could comprehend. Their laws of physics... Everything there could be different. We cannot compare what we can do here with what the "real" God-like simulators could. This would explain a lot about our random and mysterious universe. See simulation hypothesis. But even if we were really being simulated, what can we do? Life, whether simulated or not, will still continue. The universe is an enigma, indeed. Bonkers The Clown (Nonsensical Babble) 06:17, 27 December 2012 (UTC)[reply]
You still seem to be missing the fact that it's enormously harder to simulate a quantum world than a classical world. Quantization in quantum mechanics has no resemblance to the sort of discreteness you find in a computer. The state of a quantum computer is represented not by integers but by a vector in a complex vector space whose dimension is exponential in the number of qubits. Roundoff error loses data, and data loss at a fundamental physical level would violate the principle of unitarity which is an indispensable part of quantum mechanics. The idea that our world was designed for ease of simulation is idiotic. All evidence suggests exactly the opposite. -- BenRG (talk) 08:12, 27 December 2012 (UTC)[reply]
Go easy there. Both sides are making unjustified assumptions, presumably drawn from empirical experience. The so-called probabilistic nature of quantum mechanics does not suggest round-off error at all, but any observation of "glitches" (never observed) would indeed be interesting evidence in this direction. On the other side, to assume that the quantum world is inherently "too complex" for simulation, or that that the "parent reality" even has a time dimension is not justified. A simulation would only need to process the amount of information required to describe a quantum system, which rather counterintuitively does not grow by volume but rather by the surface area of a volume, thus making large systems more efficient to "simulate". — Quondum 09:53, 27 December 2012 (UTC)[reply]

@ Tevildo, it's called not the argumentum ad lapidem but the corollary argumentum I'd've slappèd 'im. And it;s hardly a fallacy. μηδείς (talk) 06:41, 26 December 2012 (UTC)[reply]

It is a fallacy, as you're not addressing the question, merely making the assertion "It's obvious that reality is real." If we define truth as "Whatever is obvious to Medeis" (or Dr Johnson), then our work here would be a lot easier, if a lot less useful. SteveBaker, as always, has provided an excellent answer - science can't address the general metaphysical question (our article noumenon covers it reasonably well) , but science can address the question "Is the world we percieve a simulation running on a computer in a world with the same characteristics as ours?" Tevildo (talk) 22:45, 26 December 2012 (UTC)[reply]
No, it's not a fallacy (not necessarily) unless one assumes that statements are only ever grounded in other statements, not physical/perceptual evidence. It's actually the request to prove reality which is fallacious, since an independent proof would have to be based on something outside reality. Aristotle recognized the futility of such questions, and I have long had the quote of him saying so at the top of my talk page. μηδείς (talk) 01:31, 27 December 2012 (UTC)[reply]
  • (Not an answer, just a verbose and speculative exploration of the question; note that some of the first part follows on The Mind's I which is the most useful reference work I can think of and spoke of replacing neurons in the brain in a vat by a computer, a choose-your-own-adventure type book, etc.) It occurs to me that if the underlying algorithm of a "brain in a vat" were understood, i.e. that the actual data being "sensed" by "consciousness" could be defined as bits of information, then you could collect all the possible sequences of bits that could be sent to it and lay them out in a numerical table of all the experiences - not just sensory information but memory and belief as well - which any conscious being could ever possibly have. If so, you have a one-to-one mapping between every possible conscious experience and a number, and the laws of physics, any physics, in any possible parallel universe we can imagine (i.e. our method of consciousness, whatever that is) can be laid out as functions which successively map a series of numbers. In this way it would appear that all experiences are real in that they are all mathematical functions which eternally and immutably exist in the sense of any mathematical function. And yet ... we have the sense that not only is there one present but one past, i.e. that one possible physics is more important than the other. Is that merely the sensation of memory and specifically memory of rules, part of the premise for a given moment? What is the difference between such a number as it exists as any number, or when it is written down, or when it is encoded in flesh? Why is the experience of neurons as they fire a kind of 'consciousness' but not the experience of water as it reaches a rolling boil? (or is it?) I suspect that that our religion of causality is not actually based on any fact, that precognition is an important and dangerous phenomenon, and that its carefully regulated expression in the brain permits the effective creation of information via paradoxes, and its time inverse permits the destruction of information, and that these processes define the one and only consciousness, which is shared between all and only seen as distinct from one person to another due to the limitations of the input data. Wnt (talk) 05:19, 27 December 2012 (UTC)[reply]

Is the ability to fart in a controlled manner a rare talent or a mundane skill?

Both Flatulist and Flatulence remain silent on the matter. Is the ability to control one's abdominal muscles like this really incredibly rare, or is it a skill which can be achieved through experimentation and practice (at least in principle) by any average person – just like, say, belcanto technique (which doesn't require extraordinary talent to master, as far as I've been told, unless you wish to pursue it on a professional level; it's just really tricky because the muscles involved are so small and numerous and your teacher can't show you directly the way they move them or guide you, the same reason why it's relatively easy to teach a chimp hand gestures/signs since you can shape their hands the way you intend them to move, but talking, no way: same problem with the muscles in your abdomen) –, but that simply hardly anyone bothers to try to learn? Perhaps the real reason why flatulists are so rare is that people believe that you "just can't learn that", you've got to be a freak of nature? --Florian Blaschke (talk) 21:50, 25 December 2012 (UTC)[reply]

Le Pétomane indicates that he had some innate ability that was developed through practice. That is, his ability to take up and expel both air and fluids through his anus was something that was inborn, and over time he developed the inate ability into an entertainment act. --Jayron32 23:03, 25 December 2012 (UTC)[reply]
So it's like being able to wiggle your ears, except much, much rarer? And you are saying that the overwhelming majority of humanity are just innately completely unable to control these anal muscles (the sphincter?) consciously (at least this precisely)? Making Le Pétomane a genuine freak of nature? OK, but how about the medieval flatulists? The quote from Piers Plowman sounds as if the level of ability a medieval flatulist needed was not all that rare, at least. So perhaps Le Pétomane is not the best example of a flatulist because his case is too extreme/unique.
By the way, Le Pétomane is already halfway there compared to the freak which Knorkator describe in their song Ich bin ein ganz besond'rer Mann, because that man is not only able to suck liquids and other objects into his anus, but even drink and eat with it. Hilarious and brilliant. --Florian Blaschke (talk) 04:11, 26 December 2012 (UTC)[reply]
I would not conclude that it innately unachievable in general. There are numerous examples of potential coordination skills that are never practised by the vast majority of people and thus seem unattainable. Those that are never practised tend to atrophy (both the neural and muscular aspects). Like squinting, winking, raising an eyebrow, moving your eyes independently (cameleonlike), riding a bicycle, concious heart rate control etc. – with dedicated effort many of these skills can be acquired later in life, and would probably be acquired naturally if taught/observed from birth or early childhood. I have heard anecdotally of a a sect/group that learns rectal cleansing using this kind of control, which would suggest that this is just another skill that can in principle be acquired by a majority given sufficient effort. — Quondum 07:05, 26 December 2012 (UTC)[reply]
You can't teach a chimp to speak because they lack the equipment to do so. StuRat (talk) 06:57, 26 December 2012 (UTC)[reply]

December 26

rare and hazardous research chemicals

Name a few chemicals that are hazardous, used rarely and applied for novel research purposes (ex:certain toxins that are researched in very few laboratories against deadly unusual microbes). — Preceding unsigned comment added by 213.120.141.254 (talk) 06:07, 26 December 2012 (UTC)[reply]

Is that what your teacher asked you to do on your homework? -- Jack of Oz [Talk] 06:58, 26 December 2012 (UTC)[reply]

nope. I am aware that advise and homeworks are not discussed here. — Preceding unsigned comment added by 213.120.141.254 (talk) 09:14, 26 December 2012 (UTC)[reply]

If it looks like homework, sounds like homework, smells like homework, it probably is homework. You've asked a question in a way that probably 1000's, even millions, of chemicals, toxins, virus particles, and other substances might qualify. And it should be easy to think of a few if you give it a moment's thought - a few more moments' thought might suggest some words to google. Why don't you just run your eye over the periodic table and look up a few promising elements? Some of the high weight elements have only been synthesied in one or two labs at considerable effort just to get a few atoms. If you narrow it down a bit what you want to class as rare, and tell us what steps you have taken so far to find out, i.e., show us you are not just lazy, we might feel more inclined to do a bit of digging for you. Floda 58.164.226.231 (talk) 11:11, 26 December 2012 (UTC)[reply]
Except that those superheavy elements are not really used for what your question seems to classify as novel research purposes – there's not much time to do much with them before they decay, given their short half-lives. Pure research is more like it. Double sharp (talk) 13:49, 26 December 2012 (UTC)[reply]
This blog[5] is a source of wonderful stories about chemicals that no sane person would handle, usually because they're explosive, corrosive, or incredibly smelly - all backed up with references to actual research that has used them. AlexTiefling (talk) 10:14, 27 December 2012 (UTC)[reply]

Bacterial infection before antibiotics

I know before the discovery of antibiotics, herbs were used to treat bacterial infection. Is there any information available about the success rate of this treatment? --PlanetEditor (talk) 09:24, 26 December 2012 (UTC)[reply]

I'm not aware of any quantitative information, but the basic answer is: dismal. No herbal treatment comes anywhere close to penicillin. For superficial infections, the most effective primitive treatments are probably either licking the wound (saliva is a pretty potent antibiotic) or spreading honey on it. Looie496 (talk) 16:18, 26 December 2012 (UTC)[reply]
Or Maggot therapy. Ruslik_Zero 18:53, 26 December 2012 (UTC)[reply]
Or putting a poultice of moldy bread on it, as the ancient Greeks had done (but that is actually a primitive form of antibiotic treatment, they just didn't know it at the time). 24.23.196.85 (talk) 01:08, 27 December 2012 (UTC)[reply]
Researching it takes me to links about how we are almost back to the pre-antibiotics era (due to superbugs). Maybe we will have to use greatpa's antibiotics again. OsmanRF34 (talk) 22:09, 26 December 2012 (UTC)[reply]

1) Is it possible to increase train speed by doubling current train width?

2) Why China reduce its train speed from 350 to 300km/h, does reducing the speed by 50km/h also increase the train safety? Why?

3) What's the main propulsion inside fast train? Electric motor?

4) With current technology, is it cheaper to build a bridge or a tunnel below sea? roscoe_x (talk) 14:51, 26 December 2012 (UTC)[reply]

I numbered your Q's for ease of response:
1) Doubling width does not automatically allow for an increase in speed. In fact, you want to minimize cross sectional area (≈height×width) to reduce drag.
2) Any decrease in speed increases safety, since it reduces the chances of a crash and also the severity.
3) There are a variety of propulsion systems used, but the best choice, IMHO, is to not have any engine on-board, and instead deliver energy using the tracks, as in a mag-lev train.
4) A bridge is cheaper in shallow water, and a tunnel in deep water (due to the height of the supports needed for a bridge). StuRat (talk) 21:42, 26 December 2012 (UTC)[reply]
1) Widening does indeed allow for faster turning.
2) Operating below maximum speed is standard practice, as it allows trains to make up time following delays.
3) High Speed Trains generally use electric motors. Delivery of propulsion from the track, as is proposed above, would incur absurdly large infrastructure costs and is not a sensible proposal for a large network.
4) A bridge is cheaper for crossing water. Building a tunnel under deep water requires a deep tunnel which is also absurdly expensive. A tunnel is used for going under mountains or the like. — Preceding unsigned comment added by 92.11.76.114 (talk) 23:38, 26 December 2012 (UTC)[reply]
1) Perhaps, if you mean wider and shorter. But wider and proportionally taller doesn't help.
3) You need to read up on maglev trains: [6].
4) The cost of building tunnels under deep water rises more slowly with additional depth than the cost of bridges does. Hence the Chunnel, as opposed to a bridge. StuRat (talk) 23:43, 26 December 2012 (UTC)[reply]
2) Per High-speed rail in China#Corruption and concerns, the official line is that "This was in response to concerns over safety, low ridership due to high ticket prices, and high energy usage." Suspicious people (like me) speculate that it is because of corruption in the construction that may compromise safety. Clarityfiend (talk) 22:21, 26 December 2012 (UTC)[reply]
1) The main problem with doubling width is that it would necessitate a change in loading gauge, and almost certainly in track gauge. High speed trains run on regular tracks at least part of the time, if only to be taken for servicing and so on. And I understand that the Chinese authorities are trying to increase the adherence to Standard Gauge throughout Eurasia, to try and let their own network link up with other nations', and thus enable truly transcontinental trains. As Russia currently uses Russian Gauge, as do many ex-Soviet nations, that's quite a challenge. Introducing a variant gauge just for high speed would certainly over-complicate the task. AlexTiefling (talk) 00:08, 27 December 2012 (UTC)[reply]
I remember that according to one of Albert Speer's books, Hitler had a plan for trains twice as large in every dimension which his underlings, citing similar concerns, politely convinced him to defer until after he won the war. Wnt (talk) 00:17, 27 December 2012 (UTC)[reply]
Stalin had no such inhibitions - see 4-14-4. Tevildo (talk) 00:43, 27 December 2012 (UTC)[reply]
1) I recommend you read up on Brunel's seven-foot gauge.
4) Unless built over very shallow water, a bridge is very limited in its maximum span compared to a tunnel, due to requirements for intermediate supports.
24.23.196.85 (talk) 01:33, 27 December 2012 (UTC)[reply]
One of the practical limits on the speed of passenger trains on existing track is the lateral g-forces exerted on them in corners. Hence the two ways to increase speeds are:
  1. To use larger radius curves - which results in difficulties with using existing track - or even with upgrading existing track without a lot of land purchasing.
  2. To attempt what British Rail did with tilting Advanced Passenger Train...which was notoriously unsuccessful.
SteveBaker (talk) 03:55, 27 December 2012 (UTC)[reply]
Is it the lateral forces on the passengers or train that's the problem ? StuRat (talk) 04:07, 27 December 2012 (UTC)[reply]
On the passengers -- they get thrown sideways. Hence the use of "pendular suspension" in many high-speed trains like the aptly-named Italian Pendolino or the Swedish X 2000. 24.23.196.85 (talk) 06:31, 27 December 2012 (UTC)[reply]

Roche limit for black holes

where is Roche limit for any black hole?Akbarmohammadzade --78.38.28.3 (talk) 16:29, 26 December 2012 (UTC)[reply]

Black holes do not have Roche limit. They are essentially inviolable. Ruslik_Zero 18:51, 26 December 2012 (UTC)[reply]
I think that answer is true, at least in a sense, but it is a bit unsatisfying to me. The Roche limit is a point where part of an object is going to fall away from it. How could something inside an event horizon fall away from the center? To illustrate, the Roche limit article says that a satellite can only be torn apart if its density is less than twice that of the primary, because otherwise the limit is inside the primary - so anything that could bring a black hole to the Roche limit would be darn near one already. And so my assumption is that the black hole + primary become a big rotating black hole. But note that a rotating black hole has a ring singularity, and so one can argue that you have disrupted the innards of the black hole and turned them into an "orbiting ring of debris" in some very, very loose sense. I'm not really giving you an answer with this, just making an appeal for an imaginative consideration by one of the experts. :) Wnt (talk) 20:32, 26 December 2012 (UTC)[reply]
This is an interesting question that I've never thought about before. There are a bunch of papers/preprints about it on the arXiv. According to arXiv:0705.1570 (which has pictures), the event horizon is distorted by the tidal force into a cigar shape. This actually surprises me, because the event horizon of a rotating black hole is not distorted: it's a sphere with no equatorial bulge. The singularity remains a point/line (I think), but the Planckian region, where the curvature exceeds the Planck curvature and therefore general relativity is presumably wrong, is flattened like a pancake. The paper mentions that there's a threshold tidal force beyond which the pancake extends outside the event horizon, giving you a naked singularity (more or less). This might imply the existence of a (mass-dependent) maximum tidal force analogous to the maximum rotational speed of a rotating black hole (beyond which you get a naked singularity). I'm not sure how this force could actually be achieved, though, since it seems to require another black hole essentially touching the first.
arXiv:0910.4311 mentions that a black hole orbiting another body will gain mass from tidal friction, and I assume (though it doesn't say so) that it would eventually become tidally locked to the other body. This isn't directly relevant to the question, but it's interesting. -- BenRG (talk) 01:34, 27 December 2012 (UTC)[reply]

thanks all. It might be two Roche limit for any black hole .One of them outer than event horizon and other inner it as we know all objects elongate infinitive there. — Preceding unsigned comment added by Akbarmohammadzade (talkcontribs) 08:52, 27 December 2012 (UTC)[reply]

It would be useful to frame the question properly. The Roche limit for a body applies in relation to another, and depends upon properties of both (the mass of the primary, and the density and configuration of the secondary). Thus, the Roche limit of a (normal matter) body orbiting a black hole (the primary) will be the radius from the black hole where the object will disintegrate under the influence of tidal forces, and this will typically be far outside the event horizon of the black hole for smaller black holes, but inside the event horizon for supermassive black holes such as in the centre of the galaxy. The radius of a black hole's event horizon is proportional to its mass. The Roche limit is proportional to the cube root of the ratio of the black hole's mass and the orbiting body's density. The previous answers assumed that the black hole is the secondary, orbiting an even larger primary. Here the (infinite) density of a non-rotating black hole makes the Roche limit zero radius from the primary's centre, though the rotating case (a Kerr black holes as the secondary) might be interesting, since the singularity presumably differs non-trivially from a spherical configuration. — Quondum 09:35, 27 December 2012 (UTC)[reply]

When we talk about black holes we imagine some thing absorbing and destroying all around it .This is not reality about them. If it is so that any black hole has infinitive mighty and effect at all distance , this has contraction with nature laws. I want to say (as said last about planets round black hole )that the black hole with mass M and rotating properties , has as gravity field as last star .we say if any black hole was any star which had planets , its members will rotate round it safely . if our sun was able to be black hole ,our earth was rotating round it ,without any difference later and after . why we do such mistake ? black hole sun cannot destroy not only low density Saturn but also near mercury ,then we give some hypothetical properties to it, of absorbing all the things? can any black hole absorb its neighborhood star?never .

have you never calculate Roche limit of any black hole?

excuse me --Akbarmohammadzade (talk) 10:47, 27 December 2012 (UTC)<[reply]

Past article from the Lancet medical journal.

I require a specific article from a past Lancet journal. The article if from 2005 volume366 pages 29-36.IT IS specifically about the ABCD criteria after a TIA (transient ischemic attack). — Preceding unsigned comment added by 24.185.248.186 (talk) 16:32, 26 December 2012 (UTC)[reply]

The article is titled A simple score (ABCD) to identify individuals at high early risk of stroke after transient ischaemic attack. If you search for the title at scholar.google.com, you can get access to several downloadable online versions, including this one. This way of finding things works pretty frequently. Looie496 (talk) 17:23, 26 December 2012 (UTC)[reply]
Wikipedia:WikiProject Resource Exchange/Resource Request is the place to request articles that aren't available free online. -- BenRG (talk) 17:52, 26 December 2012 (UTC)[reply]
How does it come that this is legal? OsmanRF34 (talk) 21:48, 26 December 2012 (UTC)[reply]
We don't give legal advice. But you might want to read fair use. 208.102.63.50 (talk) 23:04, 26 December 2012 (UTC)[reply]

December 27

measuring in a hypodermic syringe

How does one accurately measure small amounts of medicine in a hypodermic syringe? Is there medication left in the needle after the injection?Tmary (talk) 01:07, 27 December 2012 (UTC)[reply]

(1) The hypodermic syringe usually has markings on the side showing the volume of medicine to be delivered. (2) Yes, there is a small amount of medicine left in the needle after the injection, but it's unusable because it cannot be expelled from the needle, and also because it's mixed with the patient's (possibly infected) blood. 24.23.196.85 (talk) 01:16, 27 December 2012 (UTC)[reply]
2) I think what they are asking about is if the unused portion remaining in the syringe is accounted for in the markings. I'm not sure, but guess that this amount is insignificant, either way. StuRat (talk) 01:26, 27 December 2012 (UTC)[reply]
(2) All the syringes I've seen are calibrated "to deliver" -- which means that the markings show the volume actually injected. 24.23.196.85 (talk) 01:41, 27 December 2012 (UTC)[reply]
also, a standard amount of medicine is in solution by weight per volume in the liquid matrix. So one simply measures a volume of a usually pre-made solution to be injected, rather than measuring some miniscule weight. μηδείς (talk) 01:25, 27 December 2012 (UTC)[reply]
Seems like the graduations measure the amount in the barrel - so when you suck the liquid inside, you're getting a little extra inside the needle...then when you do the injection, that exact amount is left inside the needle at the end - so the reading on the barrel should be exactly correct no matter the volume inside the needle. I suspect that the limitations on precision are more to do with the skill and eyesight of the operator than anything else. SteveBaker (talk) 03:46, 27 December 2012 (UTC)[reply]
There is a small dead volume in many sorts of equipment. One quick way to avoid it being a problem is to keep its contents constant. That is, if you keep the syringe needle-down, pull the syringe barrel from the 0.0 mL marking to 0.2 mL and then push it out to 0.0 mL, you have picked up and squirted out 0.2 mL. The needle (and the inner part of the Luer lock and other areas) started empty and wind up empty. Or else you could pull to 0.3 mL, invert and push down to 0.2 mL (expelling the air that had been in the needle, etc.). Then when you push to 0.0 mL, you are still ejecting "0.2 mL to 0.0 mL", now indeed wasting the drops in the needle. Either way, you keep the same material in that space, whatever it is does not interfere when the barrel moves a certain number of volume-markings (standard laboratory practice when titrating by volume from a burette). DMacks (talk) 03:48, 27 December 2012 (UTC)[reply]
Note that having a constant 'dead volume' in a syringe depends on the contents being liquid (and specifically aqueous, as water is practically incompressible), i.e. the syringe is commonly tapped to bring any air bubble(s) to the top, and the uppermost contents, liquid and gas alike, are expelled down to the desired precise quantity. Wnt (talk) 04:40, 27 December 2012 (UTC)[reply]
Something else I should add is that the syringe should be chosen for the amount of med to be given. You wouldn't use a 100 ml syringe to administer 5 ml of meds, as it would be less accurate at that dosage than a 5 or 10 ml syringe. StuRat (talk) 04:43, 27 December 2012 (UTC)[reply]

decline of space expansion and exploration

I came across an old (pre-70's) transcript of an Earl Nightingale radio broadcast in which he spoke of Seaborg's predictions for the year 2000. Seaborg predicted that moon trips would be commonplace by 1992, and people would be able to go around the world in 2 hours, and that we would be visiting nearby planets by the year 2000. Interesting that he also predicted the 'internet'... he said that people would be able to use radio technology to read books from remote libraries :) Now my question is - why is mankind not devoting resources into space expansion and exploration? Surely, with the large number of extinction events possible, that we would need a 'backup' of mankind in space bases on the moon and beyond? Yes I know the traditional response to this is that it's 'prohibitively' expensive, but surely there's got to be more to it than this? What's the use hoarding or misusing funds if a comet can wipe us out tomorrow? Sandman30s (talk) 09:51, 27 December 2012 (UTC)[reply]

There almost certainly won't be any comets wiping us out tomorrow because we're constantly monitoring for any nearby objects of sufficient size to do serious damage. Launching anything into space is ridiculously expensive, even for low-earth orbit. Creating a fully autonomous offworld base (in orbit or on another body) is an extremely difficult task by itself; there's an absolutely vast number of ways in which it could go wrong. The combined cost of developing the necessary equipment and launching it into space would be outrageous. The chances of an offworld colony being wiped out would be much greater than the chances of the entire human race being wiped out. If we really wanted to protect the human race from mass extinction events, I'd imagine building underground cities would presently be a much better choice than colonising space. As for space exploration, we are doing that, except we're using robots instead of humans, because it's just that much cheaper and easier. --Link (tcm) 11:01, 27 December 2012 (UTC)[reply]